• 0 Shopping Cart $ 0.00 -->

JD Advising

🎉 JD Advising’s Biggest Sale is Live! Get 33% off our top bar exam products – Shop Now 🎉

Learn to study efficiently for the bar exam with our free guides on how to pass the MEE and MBE ! Plus memorize all of the MPT formats with our attack outlines !

Overwhelmed by your bar exam outlines? Our expert-crafted, highly-tailored bar exam outlines are now available for individual purchase! It’s not too late to change your approach!

how to succeed in law school, excel in law school, excel 1L year

How do I answer law school exam questions? (an in-depth guide)

Here, we give you an in-depth guide on how to answer law school exam questions. This is a step-by-step process for approaching law school exams.

Law school exams are very different than exams you may have taken in undergrad . Because they are different from the exams you are used to, you need a novel strategy if you want to answer the law school exam questions successfully and receive high scores on your law school exams. Many students do not know how to best prepare for law school final exams (in part because law schools do not teach students this!). In this post, we break down the process of answering law school exam questions into a few simple steps.

How to answer law school exam questions

Preparation for answering law school exam questions.

Make your outlines!

Before you learn  how  to answer law school exam questions and attempt to answer law school exam questions, it is crucial that you have outlines (that you have made) and that you have familiarized yourself with your outlines well enough so that you feel like you know them. Here we have an in-depth guide to outlining . We also have tips on how to outline using diagrams , and places you can find outlines online if you want to review examples of outlines.

Learn your outlines!

If you have outlines, but are struggling to learn or memorize them, you are not alone. Many students struggle with this aspect of studying. Check out this post for seven fantastic tips on learning your law school outlines, which is outlined previously in this guide!

Why do you need to outline and learn your outlines first? Because the essential tool that you need prior to answering exam questions is knowledge of the law! This is the key tool, or your secret weapon, that will help you work your way through exam questions. After all, exam questions will ask you to apply the law to a fact pattern . You cannot apply the law if you do not even know what the law is! No strategy, tool, or tip will be able to get you a high score on a law school final exam if you do not make an effort to truly know the law.

General format of a law school exam

Your law school exam will generally be a story (called a “fact pattern”). You will be expected to go through the story, sentence by sentence, and spot the issues. For example, you may have a fact pattern like this:

Anna is walking down the street, texting her friend. She is not looking where she is going. A driver, Bob, who is trying to get directions on his phone, does not see Anna and hits her. Anna falls to the ground and breaks her tailbone. Normally, the impact would not cause a broken tailbone but Anna has a brittle bone disease which causes her to get hurt very easily. Anna goes to the doctor later that afternoon. Anna fills out a form that clearly states she is allergic to various medicines. The doctor accidentally gives her medicine that she is allergic to—a medicine that Anna had listed on her form that she was allergic to. Anna has to stay in the hospital for five days to be treated for her allergic reaction. The visit for the broken tailbone cost Anna $20,000. The five-day stay in the hospital cost Anna $50,000. Discuss the issues. 

Your job will be to use the IRAC method, discussed below, to identify the issues, state the rules of law, apply them, and then arrive at conclusions for each issue spotted. You can see that the call of the question is open-ended (“Discuss the issues”) and that you are not actually told which issues to discuss. Note that your professor may give you several short fact patterns (like the one above) or may give you longer fact patterns (sometimes one to five pages long).

Step one: learn to answer exam questions using the IRAC method

The IRAC method is a commonly used method for writing law school exam answers. The letters in IRAC stands for the following words:

A = analysis

C = conclusion.

The IRAC method is formulaic. To use IRAC one essentially fills in the blanks of the following formula for each issue that one spots: “The issue is ___________. The rule is ___________. A court would analyze it as follows: [apply the law to the facts]. In conclusion, ___________ will likely win.”

The IRAC method, in its most basic form, is not the best way to answer exam questions. It is a bit too simplistic and formulaic. However, it is a great start for learning how to answer law school exam questions. Further, as you practice it, (and as we show you below) you will find yourself improving the analysis section significantly in order to write a sophisticated exam answer.

It is important that you identify most, if not all, of the issues in your exam fact pattern.  Write down the issue. Explicitly say, “The issue is whether . . . .” Note that many times, the fact pattern will not give you any hints as to what the issues are.  It will just say “Discuss the issues” or “Discuss.”  Thus, it will be up to you to spot all of the issues.

Issue-spotting is important because if you don’t spot the issue, you will not be able to apply the relevant law and analyze it (and this is what you get the most points for!). To become a good issue-spotter, practice answering exam questions and comparing them to model answers.  This will help you get an idea of the most commonly tested issues and will help you spot these issues on your final exam.

As an example, in the example fact pattern above, some of the issues are negligence, contributory negligence, foreseeability of medical malpractice, eggshell-skull rule, and joint and several liability. All of these issues should be discussed separately.

For each issue that you identify, state the rule of law that governs the issue. Make sure that you state the relevant rules of law.  Some students will write down all of the rules of law that they have learned to try to impress the professor. However, you don’t get points for these and you are wasting time writing them all down.

The analysis is the most important part of your exam answer. It is the section where you apply the law to the facts. The key to writing an average IRAC answer and an outstanding A+ answer is to develop the analysis by making lawyerly arguments on behalf of the plaintiff and on behalf of the defendant for each issue that you spot. When you get to the “A” in IRAC, ask yourself: “What would the plaintiff argue?” then ask yourself, “What would the defendant argue?” State who has the better argument. Make creative arguments, and make all of the reasonable arguments you can. This is the most important part of your essay.

When you make arguments (and when you respond) for each side, refer to specific facts in the fact pattern. Arguments can be made using the following strategies:

  • Sometimes you will argue that the facts should be interpreted one way or another.
  • Sometimes you will argue that the law should be interpreted one way or the other.
  • Sometimes you will cite policy reasons for why the case should turn out one way or another.
  • Sometimes you will argue that the traditional rule should be applied instead of the model rule (or vice versa) or that the common law rule should be applied instead of the statute (or vice versa), or that the majority rule should be applied instead of the minority rule (or vice versa).
  • Sometimes you will argue that the holding in one case should be followed rather than the holding in a different case.

Most of your arguments will be based on classroom discussion. That is, if your professor really emphasized contradicting rulings of two different cases, you would mention that in your argument section. If your professor focused instead on common law rulings vs. statutory law, you may make arguments based on that.

Even if the question says “you are counsel for the defense,” you still have to recognize (and state in your answer) arguments that the plaintiff would make. After all, the best defense lawyer will anticipate arguments by the plaintiff.

This takes your answer out of the simple IRAC format because you are not just applying the law to the facts, instead you are arguing and analyzing from both the plaintiff’s and the defendant’s perspective in a lawyerly manner.

Last, state which party is more likely to win and explain your reasoning. The conclusion answers the question, “Who has the better legal argument?” The conclusion that you arrive at is not as important as the analysis that you provide.

Do not leave your conclusion too inconclusive (e.g., “It depends on what the court decides.  Either could win.”).  A conclusion that is inconclusive is not a conclusion at all! However, do not leave your conclusion too decisive either (e.g., “The plaintiff will definitely win” or “The defendant’s argument is baseless.”).  It is best not to use extremely strong words or phrases unless they truly are appropriate (and this will happen very rarely for the big issues!). The keywords to use are “probably” or “most likely.” Say something like, “For all the reasons discussed above, the defendant is most likely to win on this issue.”

Step two: practice exam questions

It is not enough to have a good strategy for answering exam questions. You have to also practice that strategy . If you were trying to get good at painting, reading books about painting all day wouldn’t be enough—you would actually have to practice painting! It is the same with answering exam questions. It is not enough to have some skills or strategies. Instead, you have to practice those skills and strategies.

The three primary sources that you should use to practice applying what you know are:

  • (1) Exams that your professor has given students in the past (check with your professor) ***This is by far the best resource!*** Make sure to obtain model answers or sample answers if possible.
  • (2) Problems in supplements— such as Examples and Explanations or Glannon Guides . This is a great resource for practicing all throughout the semester because you can practice analyzing one issue at a time and hone in on your skills.
  • (3) Other resources that contain exams with model answers:  The best thing to do is Google, for example, “Contracts exam model answer.” You will find updated Contracts exams with model or sample answers. We also have compiled a great resource of practice exams here if you want to save yourself some time Googling it!

After you write your exam answers, always spend a long time comparing your answer to the model answer and grading your answer. This is how you will improve on your exam-writing skills. You will be actively learning the law, giving yourself feedback, identifying your strengths and weaknesses, and find yourself improving over time! If you do this regularly, by the time you get to the final exam, you will know what to expect, and it honestly won’t seem like that difficult or daunting of a task because you will have already done it so many times! I graduated as the #1 law student and got A+’s in over half of my law school classes. Practicing exams ahead of time was a huge reason  I was able to achieve that accomplishment.

If you are unsure how to compare your answer to the model answer, we recommend that you start by asking yourself these questions:

Questions to ask when comparing your answer to the model answer

  • Did I spot the same issues that the writer of the model answer spotted?
  • Did I miss important issues? Which ones? How will I avoid this in the future?
  • Did I include issues that the model answer did not include? If so, are these issues relevant?
  • Did I clearly lay out all the rules and elements of law for each issue?
  • Did I discuss laws that weren’t relevant? How can I avoid this in the future?
  • Did I make arguments on behalf of each party (where applicable)?
  • Did I analyze the problem as in-depth as the model answer did?
  • Did I spend too much time analyzing an issue that should obviously turn out in one party’s favor?
  • Did I know enough law to fully analyze the question or do I need to review my outline more?
  • Was my conclusion too vague? Was it too strongly worded?
  • Was my conclusion correct (or at least arguably correct)?
  • Did I spend too much time restating facts or conclusions?
  • Did I answer the exam in the appropriate amount of time?
  • What are my strengths?
  • How can I capitalize on my strengths and make them even better?
  • What do I need to practice more?

Commonly-asked questions on how to answer law school exam questions

What is the most important resource i can consult if i have limited time.

The best resource at your disposal is your professor’s old exams. These show you what the professor likes to test and can help you get used to their format and their exam approach. Make sure that you obtain any model answer or sample answer that goes with the exam. Some professors do not make past exams available for review. Some only offer a couple of prior exams available for review, in which case it is a good idea to seek exams using the other resources mentioned above.

What if my professor includes multiple-choice or short answer questions?

It is becoming more common to include a multiple-choice question section and/or a short answer question section on law school exams. Below we tell you how to answer law school exam questions that contain multiple-choice, short answer, or other nontraditional questions.

Multiple-choice

If your professor includes multiple-choice questions on your exam, it makes sense to get as much practice answering multiple-choice questions as you can before your exam. Do a Google search for questions or, if you have time, order books online with multiple-choice questions. Note: we also have hundreds of law school multiple-choice questions in our JD Advising study aids (which you can sample for free!) Many bar exam review books will have multiple-choice questions that cover the material in first-year law school courses.  It is amazing how much you can increase your score through practice!

Make sure that when you begin to practice answering multiple-choice questions that you go through the questions slowly and methodically. Dissect each question. After you read the question, ask yourself what legal issue is being tested and what legal rule you need to know to answer the question. If you do not know the legal rule, look it up in your outline. Lastly, go through the answer choices and explain why one is correct and why the other three are incorrect. If you complete all of the questions slowly and methodically when you practice, you will learn the legal rules better and you will also internalize the best way to approach questions on your exam. You will not fall for tricks!

Closer to your final exam, work on speed. Practice answering several questions each day and make sure you are able to answer them in the time allotted.

If you are looking for a lengthier article about how to improve your score on multiple-choice questions, see this post on how to answer multiple-choice questions : it is geared toward bar exam students but can be helpful to any students taking a multiple-choice exam!

Short answer

If your professor includes short answer questions on his or her exam, get your hands on as many short answer questions as possible. If your professor releases any past exams, focus on these first! Also, consult a supplement. Many supplements have plenty of short answer questions for you to answer. (Our law school study aids contain hundreds of short-answer questions written by top law professors!)

Combination

If your professor gives a combination of questions (some multiple-choice, some essay) spend your time answering both kinds of questions when you practice. Allocate your time based on how your final exam score is calculated. If, say, only 10% of the exam is based on your multiple-choice score, and 90% is based on your essay score, focus primarily on practicing essay questions. If it is the opposite, focus primarily on practicing multiple-choice questions!

Go to the next topic, Where can I find law school practice exams?

Seeking success in law school.

  • Benefit from personalized one-on-one tutoring by our seasoned law school tutors.
  • Explore our NEW and highly acclaimed law school study aids , available for a free trial.
  • Introduction
  • How to Succeed in Law School: Introduction and Table of Contents
  • An Overview of How to Succeed in Law School
  • Part 1: 1L Basics – What is 1L year?
  • Overview of 1L year: what classes will I take my first year of law school?
  • Why does the first year of law school matter so much?
  • What are the differences between the first year of law school and college?
  • What law school supplies do I need to succeed in law school?
  • Just for fun! Top 10 Latin words for law students
  • Part 2: How to Succeed on Law School Final Exams
  • Law school success: the theory
  • An overview of how to succeed on law school final exams
  • Step 1: Outline
  • How do I create a law school outline? (An in-depth guide)
  • When should I start my law school outlines?
  • How long should my law school outline be?
  • Where can I find law school outlines?
  • Step 2: Memorize your outlines
  • How do I memorize my law school outlines?
  • Step 3: Take practice exams
  • What are law school finals like?
  • How do I answer law school exam questions? (An in-depth guide)
  • Where can I find law school practice exams?
  • Step 4: Make an effective law school study schedule
  • What should my weekly law school study schedule look like?
  • What is an example of a daily law school study schedule checklist?
  • Part 3: Other Important Law School Skills
  • What are “cases” in law school?
  • How do I brief a case?
  • What is an example of a law school case brief template?
  • How do I speed-read cases in law school?
  • How to prepare for class in law school
  • Why is it so important to go to class in law school (and pay attention!)?
  • How do I prepare for class in law school (without wasting time)?
  • What are some law school note-taking tips and shortcuts?
  • Socratic Method
  • What is the Socratic Method in law school?
  • How do I survive the Socratic Method in law school?
  • Networking in law school
  • Is it important to network in law school?
  • Part 4: Law Student Frequently Asked Questions (FAQ) and Other Resources
  • Law student FAQ
  • Is a law school tutor worth it?
  • What are some other JD Advising resources for 1Ls?
  • Download Guide

After working with [my law school tutor], Jonathan, I earned grades high enough to land a summer associate position at a big law firm as a 1L. Jonathan was great because not only was he well versed in my 1L courses, but he also is a successful lawyer and a graduate of my law school. It was great being able to talk to someone who not only understood the rigor of my 1L classes, but also had proven tips for success in and beyond law school.

My [law school] tutor Melissa was very flexible to my needs and interests. She helped me achieve Honors grades in three of my four doctrinal courses of my 1L Fall semester. Very grateful for this opportunity.

  • Privacy Policy
  • Terms of Use
  • Public Interest

September Bar Exam, Whether To Be A Lawyer

By using this site, you allow the use of cookies, and you acknowledge that you have read and understand our Privacy Policy and Terms of Service .

Cookie and Privacy Settings

We may request cookies to be set on your device. We use cookies to let us know when you visit our websites, how you interact with us, to enrich your user experience, and to customize your relationship with our website.

Click on the different category headings to find out more. You can also change some of your preferences. Note that blocking some types of cookies may impact your experience on our websites and the services we are able to offer.

These cookies are strictly necessary to provide you with services available through our website and to use some of its features.

Because these cookies are strictly necessary to deliver the website, refusing them will have impact how our site functions. You always can block or delete cookies by changing your browser settings and force blocking all cookies on this website. But this will always prompt you to accept/refuse cookies when revisiting our site.

We fully respect if you want to refuse cookies but to avoid asking you again and again kindly allow us to store a cookie for that. You are free to opt out any time or opt in for other cookies to get a better experience. If you refuse cookies we will remove all set cookies in our domain.

We provide you with a list of stored cookies on your computer in our domain so you can check what we stored. Due to security reasons we are not able to show or modify cookies from other domains. You can check these in your browser security settings.

We also use different external services like Google Webfonts, Google Maps, and external Video providers. Since these providers may collect personal data like your IP address we allow you to block them here. Please be aware that this might heavily reduce the functionality and appearance of our site. Changes will take effect once you reload the page.

Google Webfont Settings:

Google Map Settings:

Google reCaptcha Settings:

Vimeo and Youtube video embeds:

You can read about our cookies and privacy settings in detail on our Privacy Policy Page.

JD Advising

  • Privacy Overview
  • Strictly Necessary Cookies

This website uses cookies so that we can provide you with the best user experience possible. Cookie information is stored in your browser and performs functions such as recognising you when you return to our website and helping our team to understand which sections of the website you find most interesting and useful.

Strictly Necessary Cookie should be enabled at all times so that we can save your preferences for cookie settings.

If you disable this cookie, we will not be able to save your preferences. This means that every time you visit this website you will need to enable or disable cookies again.

IPSA LOQUITUR

Writing Law Problem Questions

How to write a legal problem essay.

Law problem question essays give you an imaginary scenario. They then ask you to comment on the legal issues that arise and advise the parties. This guide will explain how to answer a problem essay with eight handy tips.

1. Read the Facts

The first step to answering any law problem question is to read the entirety of the facts you are given. Do not just jump into answering the question. Take your time and ensure that you fully understand all the issue involved in the case. It may help to highlight parts of the fact-pattern that you think are important.

2. Structuring Your Analysis

This is a mistake many law undergraduates and GDL students make when writing a scenario law essay. They look at the facts and recognise that they are similar to a case they’ve read. They will then immediately assume ‘this is like Joe Bloggs vs John Smith’, and answer the question accordingly. For this reason, examiners often complain that law essays lack coherent structure or proper analysis.

To avoid this pitfall, when answering legal problem questions you must adopt a structure .

Say you have an problem on whether or not the defendant is liable in the tort of negligence. The facts look similar to a particular case you’ve read on contributory negligence. Your first instinct is to start talking about defences. Stop . Ignore the similarity completely for now, and think. Before you can even discuss defences, you must talk about whether the defendant is liable in the first place. What does the law actually require you to establish to prove liability? Is the defendant liable? If so, what defences might he rely on, and how are those established?

Develop steps that you can put every scenario relating to that area of the law through in order. For example:

Contract Law

Contract law, signing agreements

  • Has there been an offer ?
  • Was the offer accepted ?
  • Are the terms of the agreement certain ?
  • Do the parties intend to be legally bound ?
  • Is there consideration ?
  • What are the terms of the contract?
  • Is there a breach ?
  • What kind of breach has occurred?
  • Is there a defence to the breach?
  • What are the innocent party’s remedies and options?

Criminal Law

Criminal law, holding a fence

  • Is the actus reus of the offence established?
  • Is the mens rea of the offence established?
  • Is there a relevant defence?

Negligence Law

Negligence law, stack of cars

  • Does the defendant owe the claimant a duty of care ?
  • Has the duty been breached ?
  • Is the breach a factual cause of the loss?
  • Is the breach a legal cause of the loss?
  • Is the loss sufficiently non-remote ?

This doesn’t mean you have devote a whole paragraph to every step. If its obvious that the defendant owed a legal duty, a single sentence pointing this out will suffice. If the question tells you there is a contract, simply note you will assume the contract was validly-formed. The most controversial and difficult points should be given the majority of your essay’s attention.

Nevertheless, you get credit for completing each necessary step in the order in which they arose. This is how a court of law would most naturally consider them. This method of structuring essays also stops you missing interesting elements of problem questions. You might miss these points by skipping straight to what you think is the most obvious issue.

3. Structuring the Overall Essay

The problem question may present you with multiple areas of law. For example, a defendant may have committed different kinds of crime, or liability might arise in both negligence and nuisance. Alternatively, the question may ask you to discuss the liability of several different people.

If you have to advise different parties, discuss the liability of each party in turn . Otherwise, a good rule is to  deal with each area of the law in turn . Give each area of law its own separate section. This will lead to a clearer essay structure than trying to deal with each event chronologically.

4. Structuring Individual Paragraphs

You can structure your discussion of individual issues any way which makes sense and follows a logical structure. One of the most popular ways for structuring discussions is the IRAC method. This stands for:

State the issue you are about to discuss

State the applicable legal rules (with authority or statutory references)

Discuss how these legal rules apply to the facts in front of you

State what the conclusion to the issue is based on your analysis

For example, lets say you are writing a problem question in tort law. You have a defendant who has run over a pedestrian with his car. You think he may be liable in negligence, so you start by considering whether he owes a duty of care. You would structure your analysis of this issue as follows:

The first issue is whether the defendant owes the claimant a duty of care.

If the case’s facts are non-novel, whether a duty is owed depends on the applicable precedent ( Robinson v CC of West Yorkshire Police ). It is established that road-users owe others a duty of care ( Nettleship v Weston ).

In this case, the defendant is a road-user because he was driving a car on the road. The claimant was also a road-user because they were a nearby pedestrian.

Therefore, the defendant owes the claimant a duty of care.

5. Not Enough Information Given?

What if the problem question does not seem to give you enough information to advise on the parties’ legal position? It is completely fine to write ‘the problem question does not give us enough information to determine X’.

For example, lets say you think that whether the defendant was in breach depends on how fast he was driving. The facts do not tell you how fast he was driving. Do not be afraid to say so!

However, saying that there isn’t enough information is NOT enough. You must then go on to say what information you would need to advise the party. Once you have set this out, explain how the law would apply to the facts if you had this information.

For example, you might say ‘if the defendant was speeding, he is likely in breach of his duty.’ Explain why. Then, say that ‘if he was not speeding, he is likely not in breach.’ Finish by explaining why this is the case.

This shows you are thinking like a lawyer. If a client walks in and gives half the facts, you need to be able to ask the right questions. You then need to be able to evaluate the facts, whatever they turn out to be.

6. Red Herrings

Sometimes, the fact-pattern will include information which seems salient, but actually is not. This is known as a red herring. These red herrings are designed to test your ability to pick apart relevant information from irrelevant information.

Many students assume that all of the information in the problem question must be relevant and addressed. When presented with a red herring, these students will panic and start guessing at how the information affects the outcome. Instead, the examiners expect you to point out that the information is not relevant. You can get bonus points by explaining why the law doesn’t consider this kind of information relevant.

For example, say you have a commercial law question where the owner of a painting stores it with a local art dealer. The owner then sees that the art dealer has put the painting up for sale without authority. The owner decides he will call the dealer later in the day to clear up the problem. However, he negligently forgets. The painting is later sold to a third-party.

The red herring here is the owner’s negligence. This seems like it should be relevant to whether the owner has lost property in the painting. In reality it is not: you cannot lose ownership of property because you were negligent. Because the information seems relevant, it is tempting to discuss it at length, for example by talking about estoppel by negligence. This wastes time and will not get you any marks. Rather, you should say ‘the owner’s negligence is not relevant to whether he retains ownership of the painting’. Then, move on.

7. Don’t Hedge Your Bets (and other Stylistic Tips)

If there’s one thing most law professors hate, its a phrase like ‘it seems from the evidence that there might be a possibility of supporting the argument that…’.

Confidence in essay-writing is not something that is stressed enough at school or university. When you aren’t sure, it is tempting to hedge your bets with language like ‘probably’ and ‘it might be the case’. Resist that urge. If your analysis is correct but don’t sound confident, the examiner may doubt that you firmly grasp the material. If it is not correct, saying ‘probably’ in front of the error won’t help in any case.

Other stylistic tips for writing a professional sounding essay include:

  • Avoid contractions (‘don’t’, ‘can’t’), slang phrases and other informal language;
  • Avoid the phrase ‘it is submitted that’. This kind of wording is for moots and legal debating, not academic legal essays;
  • Try to deal with only one issue per paragraph. This makes the essay less visually intimidating;
  • If simple language and short sentences get your point across, use simple language and short sentences. There is a temptation to sound ‘professional’ by using multi-clauses sentences and complex vocabulary. This just makes the essay harder to read.

8. Cite, Cite and Cite Again

If you ever make any kind of positive claim about the law, back it up with a citation. What proves your claim? A case? A statutory provision? Cite it. You need to assure the marker that you aren’t just making lucky guesses. Also, many institutions’ grading criteria specify that you can’t get more than a 2:2 if there is insufficient citation.

Generally there is no need to give the year, report and page number of case-law in exams. However, you should check your university’s best practice guidelines to know for sure.

Share this:

answering essay questions in law

IRAC

IRAC: How to Answer Law Problem Questions

You just got ushered into the examination hall and you take your seat. It’s time for another paper, and you’re hoping that you remember all that you read moments before the exams. After all students have sat down, invigilators share the exam questions and you promptly get yours.

As you look through the questions, you suddenly panic. The exam questions look like passages from a novel. You know what this means — problem questions. You read through the questions but things only seem to get worse, you don’t know what to do. You don’t know how to answer the questions. You don’t know where to start.

Problem questions can be quite problematic, if you don’t know  what to do. Luckily, they are also the easiest questions to answer — if you know your onions. The general technique that you can use to deal with any problem question is the simple formula: IRAC.

If you understand how to use IRAC, dealing with problem questions will seem less problematic. By the time you’re done with this post, you should be able to put any problem question in its place.

What is IRAC?

IRAC is simply an acronym for:

A pplication

C onclusion.

IRAC is a formula that is used throughout the broad sphere of legal writing. IRAC or slight variations is used by judges in delivering judgements, by lawyers in writing their briefs, by lawyers when giving legal opinions, and numerous other areas of law.

In essence, understanding IRAC will not only be useful for your exams, it would help you throughout your career as a lawyer.

How to Use IRAC

Now that we understand what IRAC is, we get to the most important part – actually using it.

Just like we did in the guide to answering law essay questions , I am going to give you a sample question that we will use IRAC for.

This is the question:

Mr Daniel Kiss is a seasoned Fuji musician, he heard of the 10th year wedding anniversary of his long-time friend, Mr Starboy, who wanted to celebrate it in a big way. Mr Daniel Kiss was actually at the ceremony and was delighted to have been called by Mr Starboy to perform at the ceremony. The performance was adjudged by  many people at the ceremony to be superb. After the performance, Mr Owolabi, who was highly impressed, promised to pay Mr Daniel Kiss a sum of 500,000 naira as a reward for his performance.  However, Mr Owolabi didn’t pay this sum and  Mr Daniel Kiss instituted an action to get the money. Advice Mr Daniel Kiss on the chances of success or otherwise of his action.

So, this is how you use IRAC to deal with a problem question:

When you want to determine the issue in a problem question, you have to look for the area of conflict. The conflict in a problem is where the interest of the characters clash and there is a disagreement. Looking at the scenario above, I have emphasized the area of conflict:

After the performance, Mr Owolabi, who was highly impressed, promised to pay Mr Daniel Kiss a sum of 500,000 naira as a reward for his performance. However, Mr Owolabi didn’t pay this sum and  Mr Daniel Kiss instituted an action to get the money. 

From the part I have emphasized here, the conflict involves a promise to pay for an action that has already occurred in the past. In essence, the conflict involves past consideration.

With this in mind, we can formulate the issue in this question as:

Whether or not Mr Daniel  Kiss’ performance was past consideration for Mr Owolabi’s promise?

There are some things you should notice in the structure of the issue. Note that it makes use of “Whether or not”, You can either use this phrase or “whether” when writing an issue. This  is due to the fact that in court cases, issues are usually couched in the form of questions. However, there are some lecturers who don’t want issues couched this way, and they will let you know.

You should also note that the issue relates the facts of the case with the area of law you’re considering. In essence, your issue would be incomplete if you just state something like “whether there was past consideration” or “whether Mr Owolabi owes Daniel Kiss some money”. The perfect issue is a unique combination of facts and law.

The rule is the section of your answer where you resort to authority. This can either be by stating statutory provisions or case law relevant to the issue.

The appeal to authority is something that is important to all law students. It is the provision of case law and statute that separates the writing of a lawyer from a sociologist, political scientist, or any other field of social science.

You should also try to define the legal concept that the question deals with. If the definition is something contained in statute, case law, or any other source of law , you should do well to quote it. If you don’t have any authority to quote, just give a definition to the best of your understanding.

This is an example of the rule for this question:

According to the Black’s Law Dictionary 9th Edition , past consideration can be defined as “An act done or a promise given by a promisee before making a promise sought to be enforced. Past consideration is not consideration for the new promise because it has not been given in exchange for this promise.” In the case of Akenzua II, Oba of Benin vs. Benin Divisional Council (1959) WRNLR 1 , the defendant asked the plaintiff to use his influence to convince the African Timber and Plywood company to release some forest areas to it. The plaintiff was able to successfully prevail on the African Timber and Plywood company to do this. After securing this, the Oba told the council to release part of the land to him for his exclusive use. The Provisional Council agreed to do this. However, it subsequently withdrew its assent. As a result, the Oba sued them to enforce the “contract”. The court held that the Oba’s act was not valid consideration for the Provisional Council’s promise because when the Oba carried out the act, he didn’t do it for a promise. As a result, his consideration is past and there is no valid contract that can be enforced.

Application

In the application part of IRAC, you are going to relate the authority(s) you have used with the facts of the case. This is how you should do it:

In the present scenario, Mr Daniel Kiss performed before the promise of reward by Mr Owolabi. This is similar to Oba Akenzua getting the land before the promise of reward by the Benin Provisional Council in the case of Akenzua II, Oba of Benin vs Benin Provisional Council . In this case, the court held that such act was past consideration, and it makes the contract unenforceable. In the same vein, Mr Daniel Kiss’ performance is past consideration for the promise made by Mr Owolabi. As a result, it is not a valid contract that can be enforced  by the court.

Your conclusion is the part where you give advice to the party the question asked you to give advice to. You can glimpse this from the last sentence of this question which reads:

Advice Mr Daniel Kiss on the chances of success or otherwise of his action.

So, this is how you would advice Mr Daniel Kiss in the concluding part of IRAC:

My advice to Mr Daniel Kiss is that he should not go ahead with the suit because his consideration is past consideration, making the contract unenforceable.

The full Answer

To make things clearer, this is what the full answer to the problem question would look like:

The issue here is: Whether or not Mr Daniel  Kiss’ performance was past consideration for Mr Owolabi’s promise. According to the Black’s Law Dictionary 9th Edition , past consideration can be defined as “An act done or a promise given by a promisee before making a promise sought to be enforced. Past consideration is not consideration for the new promise because it has not been given in exchange for this promise. In the case of Akenzua II, Oba of Benin vs. Benin Divisional Council (1959) WRNLR 1 , the defendant asked the plaintiff/Oba to use his influence to convince the African Timber and Plywood company to release some forest areas to it. The Oba was able to successfully prevail on the African Timber and Plywood company to do this. After securing this, the Oba told the council to release part of the land to him for his exclusive use. The Provisional Council agreed to do this. However, it subsequently withdrew its assent. As a result, the Oba sued them to enforce the “contract”. The court held that the Oba’s act was not valid consideration for the Provisional Council’s promise because when the Oba carried out the act, he didn’t do it for a promise. As a result, his consideration is past and there is no valid contract  that can be enforced. In the present scenario, Mr Daniel Kiss performed before the promise of reward by Mr Owolabi. This is similar to Oba Akenzua getting the land before the promise of reward by the Benin Provisional Council in the case of Akenzua II, Oba of Benin vs Benin Provisional Council . In this case, the court held that such act was past consideration, and it makes the contract unenforceable. In the same vein, Mr Daniel Kiss’ performance is past consideration for the promise made by Mr Owolabi. As a result, it is not a valid contract that can be enforced  by the court. My advice to Mr Daniel Kiss is that he should not go ahead with the suit because his consideration is past consideration, making the contract unenforceable.

So, here you have it, how to answer problem questions with IRAC. If you know your way, it’s quite straightforward. If there are areas that aren’t clear to you, you can leave a comment. Also, don’t forget to share this post with any of your friends that might be needing it. You can easily do that with  the share buttons on the screen. To your success!!

P.S: If you are interested in an online course that makes it easy for you to get A’s in your law exams, you can check it out here:  Get Access to Ace LL.B Exams .

128 thoughts on “ IRAC: How to Answer Law Problem Questions ”

Hi Olamide, thanks a lot for this. My question is whether or not there is a way to know how many issues are contained in a problem question?

Yes, you just have to be on the look out for multiple areas of conflict in the question.

So how do I treat them

What if I don’t know the rules to apply and cases to refer in particular scenarios

1.I find it quite difficult to identify the issues in a question.Any solution? 2.How can I differentiate a problem question from an essay question?

To know the issue in a question, you should just look for the area of conflict in the question, just like in the example. About differentiating problem and essay questions, the differentiating factor is that problem questions have a scenario/story and you have to give advice at the end.

Mr olamide thanks so I really appreciate But in the whereby you asked to discuss the concept of Accord and satisfaction with regards to consideration using case law? How would your answer look like?

That’s an essay question.

Thank you so much Sir. You have really inspired me. Sir in a situation where there are two or more issues raised,how will you go about answering the question in such a situation?

Answer the issues separately.

thanks Olamide for your help am sure to be a better student and you will be proud of me. thanks a lot once.

You’re very welcome.

Sir, when stating the rule while applying the irac method, for instance citing a case, do necessarily need to state the fact of the case or can I just present the holding of the case as a rule.

Thanks sir for sharing your knowledge and understanding with us. In the main time, sir i have a question pls. Can someone advice the parties that is involved in the scenario with only one identify legal issue? i.e you only raised one legal issue on the question and use it to advice two or three parties that are involved in the scenario or story.

Thanks sir for sharing your knowledge and understanding with us. In the main time, sir i have a question pls. Can someone advice the parties that are involved in the scenario with only one identified legal issue? i.e you only raised one legal issue on the question and use it to advice two or three parties that are involved in the scenario or story.

Yes. You can do that, if there is only one issue in the scenario, and if the question tells you to advise all parties.

please sir does this formula apply to a legal report. for instance you were made a secretary to a panel set-up to investigate a matter in report will you use the same format

Thanks graciously for this. My question is must definitions to terms in the Rule forumla b from the law dictionary??

2ndly, can we rephrase the issue to be Whether or not is Mr Owolabi’s promise was on past consideration to Mr Daniel’s promise

No. You don’t have to use the Law Dictionary for definitions, It’s just better. Also, on the rephrasing, Mr Owolabi’s promise is not past consideration. Rather it is the performance by Daniel that is past consideration.

Noted. Thank u

THANKS ,I understand it now.

thank you once again sir. Sir, i wanna ask you a question under criminal law sir. if A and B planned to do robbery but agreed that they would not carry any gun along. but on getting there A brought out a gun and B said, but we agreed not to carry any gun along. nevertheless, A went ahead and shoot C the owner of the car and he died instantly. then both A and B made away with the car and later apprehended by the police. Now sir my questions are, with statutory authorities and decided cases sir, (1) who will be liable for the murder of C? (2) both A & B will liable for the murder of C or only A who brought out the gun without the notice of B that will only be liable for the offense of murder? (3) if both of them would be liable, why would they be liable for the murder or if is only A that will be liable of the murder, why would only he be liable for the murder of C sir?

i would be so grateful if my questions could be answered sir. Thank you sir.

your the best solution of our problem as lawyer in making thanks for your lesson

Thanks very much sir I have a question what if the problem has a mutiplmu issues how do you go about it.

I’m currently working on a course that would deal with this in detail. In summary, what you should do is to use IRAC for each issue, separately.

Thank you so much sir. I believe I will be able to answer a problem question now.

Thank you so much sir…God bless you for this

Thank you sir.

May God reward you. We are ever grateful.

Hello Olamide, My question is concerning whether an introduction is necessary when using IRAC: that is, whether the portion that describes the main concept at hand can come before an identification of the legal issues.

In some circumstances, you can briefly highlight the main area of law the scenario deals with. However, the introduction would be very brief, it could just be a sentence. It should not be long and winding like the kind of introduction you’ll need for an essay question.

Thanks for your guide sir… But i wanna as one question please. When we are writing the exam, do we need to even specify the steps of answering the question on the booklet? i.e do we need to specify introduction, definition, body and conclusion?

Thanks for your guide sir… But i wanna ask one question please. When we are writing the exam, do we need to even specify the steps of answering the question on the booklet? i.e do we need to specify introduction, definition, body and conclusion?

Wow… Dear Sir, you definitely explained the IRAC better than my lecturers. Thanks a lot. However, I have a question. Is it possible to get this document in a PDF that I can download. That is, this your explanation on IRAC?

Thanks for the feedback. You can’t currently do that. However, now that you mention it, I’ll definitely work on making it possible.

Introduction is apt, although it comes in form of a concise discuss on the topic the legal issue is based.

Please how do you answer this question using the IRAC method. “Emeka, a cashier to Omenka Co-operative Society defrauded the Society of a huge sum of money. Emeka who was a first offender immediately tendered the money at the police station and begged for forgiveness. The prosecutor who was moved by Emeka’s show of remorse advised the Chairman of the Society who came to report the case to drop the charges. The Chairman however insisted on the prosecution saying that the stolen money belonged to the Society and that the Executive Committee had resolved to press case to a logical conclusion. Advise the parties. ” what rule applies and what’s the issue? I can’t seem to figure it out

It relates to private prosecution of a criminal case. Check out the case of Fawehinmi vs Akilu. If a private individual wants to prosecute a case, he needs to get an endorsement from the governor.

Thanks But what abt problem question that has more than one issue??

Kudos to you man thanks alot for this write-up

You’re welcome

It’s nice ❣️❣️

Yeah… I know 😉

Thank you for this I really appreciate.

Thank you sir am grateful

thank you for your explanation but i want to ask a question, if the question has multiple issues do you have to highlght in your answers that you are using irac i.e issue: indicate the issue rule: indicate the rule etc or you go ahead and answer the question still using irac but in an essay form

You still have to use IRAC for each issue. But it depends on the kind of question though. In most instances, you do.

please how do they use to set problem question base on internal conflicts of law and the best way to answer the questions

Thank you so much. You have explained so nicely and perfectly. It has saved my day.

Thank you Olamide

Comment thank you very much sir. I think I would have probably made more A’s if I asked earlier but not withstanding my subsequent semester would be a better one. But I would have to read it. Time after time

Thanks Sir for your nice article but pls I need a brief on identifying issues in a customary law case

Thank you so much sir i think i am improving already

If murder, rape, and not paying of tax is in a scenario, is that the conflict?

Yeah. Although it mostly depends on how the question is phrased.

Thank you so much Sir I was bad in understanding law but with this information I’m able to answer to law problem questions

I’m happy to have been of help.

Thank you so much! But I have a question.. According to Plato, Justice is giving each one his due, giving an individual that which he’s naturally fitted. However Injustice is leaving one’s fit, or part and meddling with another’s part..

What is the implication of this platonic view of a philosopher. I don’t seem to understand

I haven’t really come across this quote before. It might be better if you tell me your source, Plato might have explained further.

Thank you so much! It’s really helpful I have personal questions to ask

Okay. Would you like to send me a mail then?

That’s a good one Olamide, U can be a good and wonderful friend. Thanks alot for this.

I’m glad to have been of help.

Due to the patronage in free SHS policy introduced in Ghana in 2018/2019 academic year,Esi yaa had the following agreement with her brother who teachers at Madina boys SHS regarding her son. Esi yaa’ s son kwasi Adu is taking his final year exam at Madina boys SHS. Esi Yaa’s brother kojo Bright is Adu’ s economics teacher. At the start of kwasi Adu’s final year, Esi Yaa tell her brother that she will buy him a laptop if her son gets a A’ in economics. When the results came Kwasi Adu had an A in economics, Bright is now asking Esi to buy the laptop as promised. Consider if the agreement Esi Yaa made with Kojo Bright is enforceable by law. Please how do I go about this question.

This is concerned with the intention to enter into legal relations among family members. You can check this out https://djetlawyer.com/intention-enter-legal-relations/

Hi Mr Olamide, the case scenario a above is a very good guide/example on how to identify problem questions and possibly answer Thank you

Thank you so much sir you really helped me.

Thank you very much sir. I need a brief on how to answer this question, whether or not can an infant be in a contract ?

Hello. You can check this out https://djetlawyer.com/capacity-to-contract/

I have a question sir. What if the question ask you to advise the parties and also write a brief judgement, especially in criminal law problem question?

You can use IRAC to write a judgement.

Can one use analysis onthe part of application? Does it have any notable difference if one decides to do so?

The application is the part where you make your analysis

Hello Mr Olanrewaju, the guide for answering our law questions its quite benefit me because i dont even know how to answer because my lect gave me the AIPAC formula and looking forward is quite similar with IPAC and i think this guide its really good and i can sure that i can solve the answer. Thank your sir.

also sir is it different when we use abstract or is it still same with ipac? aipac (abstract, issues, principle. application, conclusion )

If you lecturer wants that format, then you can use it. Although, I haven’t come across it before.

Good work sir, keep doing the good work this really help me when I was undergoing my diploma in law. Okay our articles help me a lot so much so I pass my law questions perfectly. I was very impressed by your work so much so I wanted to be like you and I started researching how will I be like you. I want to have a site as you do after many research and attending lesson finally I am able to learn how to create a website like your own. Thank you very much. Bless the site sir https://www.legalresearch.com.ng/2020/03/the-injunction-on-justice-in-divine.html

Nice site you have there.

I used to be recommended this website via my cousin. I am now not sure whether or not this post is written via him as nobody else recognize such designated about my trouble. You are incredible! Thank you!

I am so lucky to find myself in this site which has helped me a lot about how to answer law essay and problem questions. I must say u are the best sir

Thanks for the compliment. We’re only doing the best we can.

thank God I am here. I just gained admission to study law and my school is about to resume. I was just engaging in researches on how to answer law questions and then I stumbled on this site. you’ve a new fan here sir, pls sir God grant you grace never to quit

Hello. Congratulations. I’m very glad to have been of help to you.

Please when there is more than one issue how do I represent it

If there are multiple issues, you have to highlight each of them. It depends on the question though, as it’s not so straightforward. This is something I addressed in my online course though.

Sir must you define all the elements of a contract when you are given a contract law questions

Sir if you are told to advice all parties in a question Will you do it one place or in a separate place. Sir is there a present consideration? If there’s one give me an example please.

Good day sir, I wanted to ask that, what does one do when he doesn’t know any rule to use in a problem question?

Ah. That is a serious problem O. As a law student, you should always be ready to back up your points with legal authorities. You should probably learn how to easily memorise legal authorities as that would help you remember rules to use in a problem question. This blog post should be of help:

https://djetlawyer.com/how-to-easily-memorise-legal-authorities/

Pls I have four questions If there are multiple legal issues,how do i answer them using irac? Also,when I’m asked to advise both parties, do I present defences to the defendants and remedies to the plaintif, or give my own opinion on what I feel they should do, with reasons? Do I call them by their names when presenting the issues or stick to plaintiff and defendant? Also, when I’m asked to advise the parties in any question,do I first conclude,as per following the irac rule or give my advice in the conclusion? Pls I anticipate your quick reply Thank you?

Please how do I get to know different cases that could be linked to particular problem question..learning the m is a tough one. I NEED HELP!!!

How do I write/answer this question with relevant cases? Agatha and Ben have been investing on the Zimbabwe stock exchange for 20years with the assistance of their Investment advisor Masimba. Masimba has access to Agatha and Ben’s investment account which is worth 5 million dollars. Masimba approached XYZ bank and opened two bank accounts in the name of Agatha and Ben and made himself a signatory to both accounts. Masimba transferred over 3.5million dollars from the investment account into Agatha and Ben personal accounts and eventually withdrew the funds from both accounts.Upon requesting their statement Agatha and Ben observed the anomaly and they are now suing the bank for negligence. Sighting a relevant case does Agatha and Ben have a legal case?

How do I answer a problem question, if it has more than one issue? Will i take the issue one by one and provide solutions to them, or I’m just going to answer it together

Depends on the question. But most likely, you have to address the issues individually.

Thanks a lot..sure this will really help

thank you, sir

Thanks so much for this

Thank you so much for this post, My question is what if you’ve been given a scenario like the one above and instead of question on advice the party’s, a direct question is given. Like they ask as pertaining to your scenario what is past consideration, or explain it in relation with something else. Is IRAC still advisable to be used.

What if the case is based on argument with respect to cult activities in a particular community..what will be the right issue to present in answering such problem?

Thank you, this is very helpful. Please, can you write on how to identify issues in courses, notably criminal law, torts and human rights?

Sir I understand your teaching on how to use IRAC to solve a problem question and am clear about that. But my challenge is how I can differentiate between problem questions and essay please I need ur help dearly.?????? Thanks Sir.

I kindly need help on this

Agatha and Ben have been investing on the Zimbabwe stock exchange for 20years with the assistance of their Investment advisor Masimba. Masimba has access to Agatha and Ben’s investment account which is worth 5 million dollars. Masimba approached XYZ bank and opened two bank accounts in the name of Agatha and Ben and made himself a signatory to both accounts. Masimba transferred over 3.5million dollars from the investment account into Agatha and Ben personal accounts and eventually withdrew the funds from both accounts.Upon requesting their statement Agatha and Ben observed the anomaly and they are now suing the bank for negligence. Sighting a relevant case does Agatha and Ben have a legal case?

I really appreciate your efforts in helping law students solve contending legal issues. Now my question is this scenario or in another where there are exceptions, how then is the best possible way to answer the question?

Thanks for ur properly guidance

Two students, who were in an electoral body, were involved in electoral malpractice and were rusticated. Your chamber has been approached to prove to the school authority that the issue of the student election is purely a student matter and should not be a curricular issue. Is this a problem question?

This not a problem question simply because there’s no conflict between both parties

Please why are you not serving ads here? It’s a really good job you do here!

Thank you so much Mr Olanrewaju,this is so helpful and explicit.God bless you.

I wish to ask some questions Sir🙇‍♀️ 1.what should I do for in case I omitted any party to the case (either the plaintiff’s name or the defendant’s name) or both,but the facts and court holding are known

2.Or if the the principles are clearly understandable but can not be quoted as they are…?

Nice and informative post sir. This gives me clarity on how to approach legal questions. Thanks once again Mr. Olamide!!

Thanks alot

AM sir Olamide please I would be delighted if u can help me with how to go about questions like this Question😎

_The National Assembly has passed a bill on electronic voting system. The bill has successfully passed through all the law making process only remaining the president assent. The president who has being with the bill for over 35days has refused to sign the bill into law. He didn’t give any valid reason for withholding his assent. He only said that the bill can only becomes law if he wish_

_A.With the aid of statutory provisions and judicial authorities, discuss the legal issues arising from the above case scenario._

_B. Can the bill be passed into law even without the president assent? If your answer is in the affirmative, state reasons with statutory provisions and judicial authorities._

Thank you so much sir. This has really helped . I have been using IRAC wrongly

Kindly help me with all the issues. Thanks, After 15 years of trade in the cocoa industry, Madam Echoke decided to expand into 5 different African Countries. She therefore engaged the services of Kwaku Frimpong to trade from Ghana to Nigeria. After three (3) successful trips to Nigeria, Kwaku Frimpong was promoted to handle all 5 African countries. There was a 40-footer container to be shipped to a new client in Rwanda. While the cocoa beans were being transported to Tema Harbour, the truck got stuck for 2 weeks. Finally, the cocoa beans were on board to Rwanda via Maersk Shipping line. The shipping line noticed during the voyage that the cocoa beans were becoming moldy. Discovering that the cocoa beans will greatly devalue before getting to Rwanda, the shipping line got an offer and sold the cocoa beans at Ethopia, after several impossible calls to Kwaku Frimpong. The new buyer paid the agreed price (based on current market value) to Maersk Shipping line’s transactional account which was immediately transferred to Kwaku Frimpong. Kwaku Frimpong under-declared the said payment and paid Madam Echoke. Advice Madam Echoke and Kwaku Frimpong.

Thanks alot Mr. Olamide. I’ve always had a problem with answering problem questions but you’ve just explained it in a more comprehensible manner. Thanks!

Please sir, how do I know the type of question to apply FIRAC?

You mean IRAC? You apply it to problem questions. These are usually questions that have a scenario. Although, not all questions with scenarios are problem questions, most of them are.

I find this piece very helpful.

Thank you very much.

Your words have a way of calming my anxieties and soothing my soul.

I have been seeing IRAC everywhere but no one has broken it down for me to understand it like this article. Thank you so much for sharing this knowledge i know it will be of good help for me and many others.

Please can you use a rule like criminal law or theories of law to give more examples

John,Emeka and Chinedu are students of the faculty of law Godfrey okoye university.John was in first year while Emekawas in second year One particular Friday night,John and Emeka returned from a disco party at about 3am and switch on the light in their room and began a hot argument about events in the faculty since they resumed.John complained about the nature of legal method saying “The course is too dry and technical”.Emeka agreed with John and frankly admitted that he was constrained to cram the course when it was giving him too much problems.He however quickly added it with a sense of pride that he ultimately scored 60% in the course Chinedu who was already fast asleep was awoken by the argument of John and Emeka.Chinedu reminded the Duo of the universities regulations that all rooms light must be switched off by 12 midnight to allow the occupant to sleep in peace.When his advice was ignored, Chinedu angrily called them a “lawless lawyer”.John who was a member of a wrestling club in his secondary school days, pulled Chinedu out of his bed and beat him to a pulp.Chinedu vowed to retaliate . Unknown to john, Chinedu is the leader of campus secret cult called “Aye boys”.Advise the parties using IRAC stating the issues,rules, application and conclusion one by one within the legal method framework and the issues separately with their rules and application

Thanks lawyer for the explanation on the IRAC. I’m reading Business Law this semester but I always find it difficult to understand basic principles and I could do well in midsemester exams. I really need your assist.

Thank you sir I saw exactly this question in my exam Other questions like Lapse of Time and Promisory estoppel too

Leave a Reply Cancel reply

Notify me by email when the comment gets approved.

Join an online course that makes it easy for you to get A’s in your law exams, you can check it out here: Get Access to Ace LL.B Exams.

How to Answer Law Exam Questions perfectly (step by step with examples)

Isack Kimaro

  • 18 January, 2023

How to Answer Law Exam Questions (with examples)

This is a definitive guide on how to answer law exam questions.

This guide will tell you how to answer essay-type questions and scenario/problem questions in a law exam with sample questions/examples.

I know, the process of becoming a lawyer/ an advocate  might become hard when it comes to law exams.

See: How to become a lawyer: From Undergrad to the Bar

Generally, law exams are technical and demand a high degree of attention and care when attending.

Transform Your Communication, Elevate Your Career!

Ready to take your professional communication skills to new heights? Dive into the world of persuasive business correspondence with my latest book, “From Pen to Profit: The Ultimate Guide to Crafting Persuasive Business Correspondence.”

from pen to profit 3D mult

What You’ll Gain:

  • Proven techniques for crafting persuasive letters, emails, and proposals.
  • Insights into tailoring your messages for different audiences.
  • Strategies for overcoming objections and turning challenges into opportunities.
  • Real-world examples of successful business correspondence.

According to the National Conference of Bar exams (NCBE) , the lowest percentage of overall pass rates for the February 2022 bar exam as reported by each jurisdiction following the release of their exam results in the US was 52.6% ( Alabama )

The focus here is to give you basic tips that will act as a guide when doing law school exams , bar exams , or any other law examination so that you can be among those 52.6%.

Here I will take you through;

  • Types of questions in law exams
  • How to answer Essay-type questions in a law exam
  • Sample answer to Essay-type question in a law exam
  • Scenario or problem-type questions in a law exam (IRAC)
  • Sample answer to Scenario type question
  • How to study for law school exams (and get passed)
  • How to study law and pass (pro tips for beginners)

Jump to section

Types of questions in a law exam

Essay-type questions in a law exam, example of essay-type questions in a law exam.

Introduction

Example of scenario or problem type question in a law exam

How to answer the scenario/problem question in a law exam, rule/law applicable, arguments/application of the law to the facts.

Generally, there are two common types of questions in law exams i.e. Essay type questions and scenario/problem-type questions. Each kind has its own mode and style of answering.

Essay-type questions are the form of questions that require the candidate to explain, discuss or comment on the specific legal issues as asked by the examiner.

  • All confessions are admissions but the reverse is not the case. Discuss.
  • Copyright law does not protect an idea, it protects the expression of an idea.”  With relevant examples discuss the above statement
  • Trace the origin and development of International law

How to answer essay-type questions in a law exam

In Answering essay-type questions the candidate will be required to write an essay. An essay consists of three major parts that are an introduction, a main body, and a conclusion. In order to successfully answer questions in law exams; a candidate must exactly know what the question demands.

A student may determine the demand of the question when concentrating on the wording of the question.

The words like elucidate, critically discuss, etc. connote what is supposed to be done.

This is a very important part of the essay.

The way you mold it may tell the examiner if you understood the question or not.

Many students think that introduction is a part that contains a definition of key terms, however, that may be true but it is not an effective way of molding your introduction.

For your introduction to appear more professional do not defines terms in the introduction, rather tell the examiner how you are going to tackle the question. Show him what, how, and where you will focus when responding to the question at hand.

For the purpose of answering the exam, A good introduction contains at least 5 sentences or two paragraphs.

A good introduction must show the examiner that you have understood the question.

Example of a good introduction

Trace the origin and development of International law.
Every society is required to have relations with other societies, and in order to regulate the relations between the states, there is required a system of law thus international law. This work will focus on tracing the origin of international law and the development of international law by tracing it through its divided stages of the period which was seemed to develop from, that is during the primitive and ancient period, the Greek era (6th Century BC), the Romans era, the Middle age development, during the 15th and 16th centuries, the modern international law (Hugo Grotius), during the 19th century, during First and Second World War and thereafter and lastly on the present status of international law.

Definition of terms when necessary will come as the second part of the introduction.

Before jumping to the main body you must provide transitional sentences.

The sentence shows what you’re going to do on the main body depending on what you have been asked.

An example of a transitional sentence is ‘ The following is the origin and development of international law ’

After having your introduction, now you’re moving to the main body.

The main body contains the main answer to the question.

Your main body must be well arranged, number your points, make them visible, be clear in your explanation, and be neat.

Examiners have a lot of papers to mark, make him interested in yours by clarity, brevity, and lucid explanation. KISS (Keep it Simple and Straight) your main body.

Do not bring new issues to the conclusion. This part should contain a summary of key issues that transpired in the main body.

Scenario or problem-type questions in a law exam

Scenario or problem questions in law exam are the types of questions that contain a set of hypothetical or real facts on a certain legal issue and requires a candidate to analyze the fact and solve the problem by applying the relevant legal principles to the fact.

Scenario questions may demand the candidate to advise and/or draft a reasoned legal opinion , prepare necessary legal documents, etc. basing on the given facts.

Jane has been living with Ally for the past 8 years. They have not gone through any formal marriage ceremony. The two have been blessed with 2 children, Peter (6) and Mary (3). Jane tells you that before they started living together, Ally (a Muslim) had agreed that he would change his religion so that the two would go through a formal Christian marriage ceremony. 8 years have passed now and, although the two children of the relationship were baptized and regularly attend Christian services with their mother, Ally is unwilling to change his religion. Jane feels that she has been cheated and that it’s an embarrassment to her parents, who are very devout Christians. They have a house on a plot of land which Jane had purchased before they started the relationship. About 90% of the construction costs for the house incurred by Jane since Ally has no formal employment . Jane wants to end the relationship  and have the house registered in the name of the two children. Advice Jane accordingly or Write a concise legal opinion to Jane

Scenario or problem questions in law are answered using a simple formula called FILAC or IRAC which stands for Facts, Issues, Law applicable/Rule, and Arguments/Application of the law to the facts and  Conclusion. To successfully answer the problem question in law you must arrange your answer in FILAC or IRAC order.

How to answer scenario/problem questions in law exams/bar exams, how to answer law school exam questions, How to Answer Law Exam Questions, how to answer law exam questions sample, sample law school exam questions and answers

Example of IRAC or FILAC exam answer

The following is how you can use IRAC or FILAC method to answer a scenario or problem question in a law exam.

Here you should state the material facts of the scenario given.

If the scenario is too long and for the purpose of the time you may skip this part and move on to the next step.

From the above example, the following facts may be extracted.

  • That Jane has been living with Ally for the past 8 years.
  • That they have not gone through any formal marriage ceremony.
  • That they have been blessed with 2 children, Peter (6) and Mary (3)
  • That they have a house on a plot of the land which you had purchased before they started the relationship. About 90% of the construction costs for the house are incurred by you.
  • Jane wants to end the relationship and have the house registered in the name of the two children

NB. When extracting your material facts from the question avoid unnecessary repetition and you should consider only facts that have legal implications.

After sorting your facts move on to frame your issues.

Issues refer to the things which require a legal response in your scenario. Avoid having a lot of issues. Focus on the major issues only. From the above example, the following issues may be raised.

Whether Jane can end the relationship?

  • Whether the house can be registered in the name of the two children?

Here you should list all statutes or case laws that are applicable to the issue raised.

For example from our issues, we can see that the issues are based on marriage and land respectively

Therefore you have to list all marriage and land laws that are applicable to your scenario. for Example Law of Marriage Act, The land Act, etc. When necessary show the relevant provisions.

Here is where you’re going to attend to the questions.

When addressing the issue your arguments must be backed up by the legal authority. You should go straight to the point. And provide a reasoned conclusion at the end. Consider the example below

The law that governs conjugal relationships in Tanzania mainland is the Law of Marriage Act [CAP 29 R.E 2009]. Under section 160 (1) its provides that where it is proved that a man and woman have lived together for two years or more, in such circumstances as to have acquired the reputation of being husband and wife, there shall be a rebuttable presumption that they were duly married. According to the case of  John Kirakwe V Iddi Siko 1989 TLR 215 (HC) the only three important elements to constitute a presumption of marriage are: (a) that the parties have cohabited for over two years; (b) that the parties have acquired a reputation of being husband and wife; (c) that there was no formal marriage ceremony between the said couple. In the given scenario, element (a) and (c) are clear in the sense that you have been living with Ally for 8 years and they have not gone through any formal marriage ceremony. Element (b) is uncertain. Its certainty is depending on how the neighbors perceived your relationship. Thus if neighbors perceived your cohabitation as husband and wife, then the marriage will be presumed. But if not, the presumption will be rebutted. However in the case of Hoka Mbofu V Pastory Mwijage 1983 TLR 286 (HC) it was stated that “Where there is no allegation of presumption of marriage, section 160 of the Law of Marriage Act, 1971 cannot be invoked merely on account of concubinage association.” Therefore to end the relationship, Jane must allege presumption of marriage so as the court to invoke section 160. She can do this by formal application (petition)  to the court seeking for dissolution of marriage or separation stating the evidence regarding the conduct and circumstances of their relationship. Even when court  rebut the presumption, in the case of  Hemed S Tamim V Renata Mashayo 1994 TLR 197 (CA)  it was stated that “where the parties have lived together as husband and wife in the course of which they acquire a house, despite the rebuttal of the presumption of marriage as provided for under section 160(1) of the Law of Marriage Act 1971, the courts have the power under section 160(2)of the Act to make consequential orders as in the dissolution of marriage or separation and division of matrimonial property acquired by the parties during their relationship are one such order;” Evidently, in light of the above discussion, Jane may end the relationship.

Repeat that process for every issue you have raised.

This is the last part of your question. This part contains the general response to the whole scenario.

For example, if you have been asked to give legal advice then this part should contain specific legal advice.

Also, you may put any further remarks.

Hope you have found this guide useful.

  • 6 ways to become the best lawyer
  • 5 Incredible advocacy skills and how an advocate can archive them

Isack Kimaro

Isack Kimaro

Isack Kimaro brings over 7 years of extensive experience in professional writing. My career has been dedicated to mastering the art of clear, effective communication, essential for successfully professional correspondence.

Related Posts

wedding congratulation letter, wedding congratulation letter sample

Wedding congratulation letter 2024 [samples]

  • 3 February, 2024

Lawyer stole my settlement, what happens if a lawyer steal my settlement, lawyer steal settlement

Lawyer stole my settlement: Your legal options explained

  • 2 February, 2023

Private investigator and detective, difference between private investigator and detective

Private Investigator and Detective 2024: Who Does What and How?

  • 25 January, 2023

answering essay questions in law

answering essay questions in law

Public Law for Everyone

by Professor Mark Elliott

Writing a Law essay? Remember to argue!

Providing advice in the abstract about how to write Law essays is difficult because so much depends on the nature of the question you are answering. It’s also important to take into account whatever are the expectations for your particular course, degree programme or university. Nevertheless, a useful rule of thumb, I think, is that a good Law essay will normally set out and advance a clear thesis or argument . (Note that I’m referring here to essays as distinct from problem questions: the latter call for a different approach.)

The need for an argument

Some answers explicitly call for this. Take, for example, the following essay title:

‘Do you agree that parliamentary sovereignty is the most important principle in the UK constitution?’

Here, the question itself in effect advances an argument — that parliamentary sovereignty is the most important principle in the constitution — and invites you to say whether you agree with it or not. And in saying whether you agree, you need to advance your own argument: ‘I agree with this because…’. Or: ‘I disagree because…’. Or even (because if the question advances a position that you think implies a misconception, oversimplification or false premise, you can say so): ‘I will argue that the question oversimplifies matters by assuming that a particular constitutional principle can be singled out as uniquely important…’

Other questions may indicate in a less direct way the need for you to put forward your own argument. For example:

‘“Parliamentary sovereignty is the most important principle in the UK constitution.” Discuss.’

Here, we don’t have a ‘do you agree?’ prompt; instead, we have the apparently less directive ‘discuss’ prompt. If we read the question literally, it may seem that there is no need for you to put forward your own argument here. After all, it’s possible to ‘discuss’ something without advancing your own argument about it: you could make various points, explain various matters, and leave the reader to make up their own mind. But while this may be formally true, it’s unwise to read the question in this way, because it creates the risk that you will end up writing something very general and descriptive on the topic without going any further.

To summarise, then, there are at least three reasons for making an argument part of your essay. First, the question will often call for this, whether explicitly or implicitly, such that you wouldn’t be answering the question if you didn’t set out and develop an argument. Second, if you don’t impose on yourself the discipline of articulating and defending an argument, you risk underselling yourself by writing something that is descriptive and meandering rather than purposefully constructed . Third, setting out and developing an argument involves taking ownership of the material. By that, I mean using the material in a way that serves the purposes of your argument, showing that you are in command of it and that it is not in command of you. This, in turn, provides an opportunity to demonstrate a level of understanding that it would be hard to show in a descriptive essay that simply wandered from point to point.

Setting our your thesis

If putting forward an argument is (often) important or necessary, how should it be done? There are no great secrets here: the formula is straightforward. You should begin your essay by stating your thesis — that is, by setting out what it is that you are going to argue. This should be done in your introductory paragraph — by the time the reader reaches the end of that paragraph, they should be in no doubt about what you are going to argue. Imagine, for instance, that you are presented with the following essay title:

‘“The courts have expanded their powers of judicial review beyond all acceptable constitutional limits in recent decades; it is time to clip the judges’ wings.” Discuss.’  

In response to such a question, it might be tempting to say in your introduction that (for example) you are going to ‘show’ how the courts’ powers of judicial review have grown, ‘consider’ why this has happened and ‘examine’ the criticisms of judicial over-reach that have resulted. These are all perfectly sensible things to do when writing an essay on this topic, but if that is all you say in your introduction, you will leave the reader wondering what you think — and what you are going to argue . In contrast, an introductory paragraph that lays the foundation for essay that properly advances a thesis will set out what that thesis is. You might, for instance, take each of the propositions set out in the question and stake out your position:

‘In this essay, I will argue that (a) while the courts’ powers of judicial review have grown in recent decades, (b) it is misguided to suggest that this has breached “all acceptable constitutional limits” and (c) that those who now advocate “clip[ping] the judges’ wings” misunderstand the role of the judiciary in a rule of law-based constitution. In other words, the courts’ judicial review powers are entirely appropriate and those who seek to limit them risk undermining the rule of law.’  

An introduction of this nature would achieve two things. First, it would make clear to the reader the position you proposed to take. Second, it would immediately lend the essay a structure.

Developing your thesis

Once you have set out your thesis in the introduction, you need to develop or defend it. This will involve making a series of connected points in successive paragraphs, each of which relates to your overarching thesis. One way of thinking about this is that the individual points you make in the main body of the essay should all relate or point back in some way — and in a clear way — to the position that you staked out in the introduction.

In the example introduction above, the overarching thesis is set out in the second sentence; the individual and connecting parts of the argument are set out in propositions (a), (b) and (c) in the first sentence. One approach, therefore, would be to divide the answer, once the introduction has been written, into three parts, dealing in turn with points (a), (b) and (c). Naturally, as you work through the various parts of your argument, you will need to cite relevant evidence (cases, legislation, literature and so on) in support of your argument. You will also need to deal with matters that appear, at least at first glance, to sit in opposition to your argument (on which see further below) or which, once properly considered, require your argument to be refined.  

A key point, however you proceed, is that the reader should also be clear about how each successive point relates not only to the previous point but also to the overarching argument. The reader should never be left wondering ‘Where does this fit in?’ or ‘Why am I being told this?’ A simple way of avoiding these problems is to signpost , by saying at the beginning of each section how it relates to the overall argument. The flipside of this coin is that you should avoid saying things like ‘Another point is that…’ since this gives the impression, rightly or wrongly, that the various points in your essay have been thrown together in a random order, with little thought as to how they fit together or relate to your overall argument. Even if that’s not the case, you don’t want to risk giving the reader that impression.

A one-sided approach?

The advice set about above might seem to imply that I’m suggesting you write one-sided essays — in which you set out points that support your argument while ignoring those that don’t. However, that’s not at all what I’m suggesting. In order to set out your argument in a persuasive manner, you need to deal both with relevant points that support your argument and with relevant points that appear to challenge your argument — and, in dealing with the latter points, you need to show why they do not in fact fatally undermine your argument. In other words, the approach I’m suggesting here doesn’t mean that you should adopt a blinkered approach, paying no attention to counterarguments: rather, you need to deal with them in a way that shows that, having thought about and weighed them in the balance, you are in a position to show why your argument stands in spite of them (or why your argument can be adapted in a way that accommodates such points).  

All of this points towards a further matter: namely, that advancing an argument in your essay does not mean that you need to (or should) be argumentative in the sense of adopting a strident tone that brooks no debate or compromise. Rather, advancing an argument in the way I’ve suggested here means being thoughtful and persuasive : taking the reader with you on a journey that demonstrates that you have looked at the relevant material, carefully thought through the issues raised by the question, and arrived at a view that you are able to justify and defend through well-reasoned and suitably evidenced argument.

So what about your conclusion? If you’ve followed my advice above, it should more or less write itself. People often agonise over conclusions, perhaps thinking that there has to be some ‘big reveal’ at the end of their essay. But there doesn’t need to be — and indeed there shouldn’t be — any big reveal. There should be no surprises at the end precisely because you’ve set out your argument at the beginning and spent the rest of the essay carefully constructing the different strands of your argument. The conclusion is an opportunity to draw those stands together, but no-one should have to wait with bated breath for the conclusion before finally realising: ‘Ah, so that’s what they think!’ If that’s the impact of the conclusion on your reader, it means there’s something wrong with the introduction!

This post was first published on The Law Prof blog . It is re-published here with permission and thanks.

' src=

  • Already have a WordPress.com account? Log in now.
  • Subscribe Subscribed
  • Copy shortlink
  • Report this content
  • View post in Reader
  • Manage subscriptions
  • Collapse this bar

answering essay questions in law

How do I answer problem questions?

answering essay questions in law

Virtual Law Fairs: Making a Good Impression

answering essay questions in law

What Is A Personal Injury Lawsuit?

answering essay questions in law

  • News & Articles
  • Surviving the BPTC
  • Surviving the LPC
  • Tips for Students
  • law answered
  • law students
  • problem questions

answering essay questions in law

Whether you’re on your LLB, GDL, PGDL, CILEX, LPC, SQE… You get it – learning how to deal with problem questions will be essential to your legal education. And to be fair, it’s genuinely useful to learn how to assess problem scenarios as you’ll need to use similar skills in your professional future.

Problem questions will present you with a factual scenario and typically require you to assess the situation from a legal perspective or advise a fictional client on their rights and liabilities.

The best way to answer problem questions is to learn the technique . Examiners will expect you to answer problem questions in a particular way. Like with any type of exam, honing your technique is your best bet for problem questions too. In the case of problem questions, it’s all about learning to apply your knowledge to the scenario. So, here’s a few tips:

Use the right guides: If you know that you’ll be assessed by way of problem questions, then you should learn and revise the law in the same way. That way, the technique will be second nature to you. Reading a typical set of notes won’t help you to apply your knowledge, so we recommend using Law Answered’s notes . Their LLB and GDL notes deliberately use problem question structures (and other exam answer outlines) in order to help you learn the subject in the same way as you’ll need to apply it.

Think about “ IRAC ”:  You might hear this term from a few of your tutors. It is an acronym to describe how you should structure your answer:

  • I ssue – simply state the specific legal issue or question;
  • R ule – explain that specific legal issue and the relevant rules in detail;
  • A pplication – apply the rules to the factual scenario in the question; and
  • C onclusion – conclude: what is the end result?

For example, if you were answering a problem question on a potential murder, you would begin with a brief description of the case of R v Defendant , in which the defendant killed the victim by stabbing him. You would state that the issue is whether the defendant has committed a murder, and legally define “murder”. You would then go through the rules of the actus reus and mens rea of the offence. You would then look at the rules in detail and apply them to the particular facts of the case, using examples from common law to illustrate why (or why not) the defendant has committed a murder. You would also look at the details of any defences that might apply. You would then conclude as to the defendant’s prospects. You can find a free PDF sample guide to answering a GDL murder exam question here .

Look out for red herrings: Examiners will sometimes throw in odd details to put you off. A common trick is to include a very specific date for when something happened. This could be the day before or after a particular statute became law. If so, your entire answer will change. Another trick is to have the fictional character or client in the question make an accusation or start a lawsuit along a particular legal line which is in fact the wrong line of enquiry.

Be practical: Answering a problem question is very different to answering an essay question. This is not an opportunity to engage in academic debate and jurisprudence. Instead, imagine that you are a lawyer giving practical advice to a real client – as if you are explaining the law and evaluating their circumstances and what the outcome in real terms will be. Showing off will not help you.

Slow down! Don’t make the mistake of skim-reading the question and rushing to write your answer – you might go down completely the wrong track. A good rule of thumb is to take five minutes at the start of your exam to properly read the question and understand the key facts.

Make a plan or a note of the key facts . Try writing down a few key words, a mini mind-map of connections or a mini chronology (timeline) of events – whichever works best for your style of learning. Writing down a few key words or facts will help you digest the problem question. For the same reason it’s good to occasionally check back in and re-read the question as you write your answer.

Answer the question set! Most importantly, you need to make sure you are answering the exact question, with the exact facts, that you have been set. So many students slip up because they do not check back in with the question and end up side-tracked and discussing a marginally relevant issue. Your answer is only worth any points if it is directly relevant to the problem question scenario. Look back to make sure that you are directly answering the exact question set. Use the same terminology that the question uses to avoid straying.

Now, if that all makes sense, you need to make sure that you’ve got the technique straight in your head. The solution is simple: practise, practise, practise! Take every opportunity you can to practise. Put effort into any mock exams on your course and take any feedback on board. Get hold of as many past papers as you can and practise writing answers under timed conditions. If your tutor is nice, they may be amenable to you asking for extra feedback. Never be afraid to ask for help as early as you can! It would be far worse to wait until the last minute.

Camilla Uppal

Camilla Uppal

Related posts.

answering essay questions in law

The Importance of Having a Life Outside of Law

answering essay questions in law

Promoting Diversity and Inclusion in Legal Tech

A man inside a clock

Review on the billable hour

Leave a reply cancel reply.

Your email address will not be published. Required fields are marked *

Save my name, email, and website in this browser for the next time I comment.

How to Answer Law Essay & Problem Questions Perfectly

  • Post author: Edeh Samuel Chukwuemeka ACMC
  • Post published: June 18, 2024
  • Post category: Legal Articles / Scholarly Articles
  • Post comments: 30 Comments

How to answer law questions: Over the years, law schools and law universities have evolved two methods of asking test or exam questions. They are the problem and essay questions patterns. Basically, these two ways of asking questions require a totally different ways of answering them that are different from the traditional ways of answering questions in the primary, secondary, and tertiary institutions.

One major problem that new law university intakes encounter, is the fact that many of them usually have the mindset that where they are (university) will be the same with where they are coming from (secondary school); and because of this, they tend not to ask questions about how well to answer their law questions thus going on to use the traditional or general method and ending up not getting the expected result or even failing.

In this article, I am going to be expounding on the way of tackling law essay questions for the benefit of students. However, what I will be discussing is the general rule which is subject to what your specific tutor or lecturer may require from you.

So, it is advised that in as much as you learn how to generally answer law essay questions, you should pay attention and understand your lecturers so as to give him/her what their individual preferences are, like asking you to write a concised answer or be elaborate, or add this and remove that  etc.

  • Differences between cross-offers and counter-offers
  • Hierarchy of courts in Nigeria
  • Mojekwu v Mojekwu: Facts, Issues and Decision of the court

Table of Contents

Differences Between Law Problem Questions and Essay Questions

As you may know, Law problem questions are not the same as essay questions. Problem questions are those law questions that tell live stories about the relationship between people and then require you to identify legal issues from those interactions, address the issues with relevant authorities and then to advise the parties on their different rights using the IRAC method.

It is important to note that a single problem question could have a series of different events which are based on different legal principles.

On the other hand, an essay question is a question which requires an individual to write on legal principles without using a particular format like the IRAC method.

Notminding the fact that you are not required to advise the parties or to use a particular method to answer your question, you have to have some mental steps at the back of your mind so as to make your work enticing and arranged before your tutor or Lecturer.

How To Answer Law Problem Questions (Using IRAC) 

Just like I explained at the beginning of this work, a problem question is a question that test candidates by giving them stories/cases to solve. Here, law students will be expected to draw out the issues of law in the story, as it relates to what they have been taught in the classroom.

Take for instance, a problem question on customary law might tell the story of a man who beats his wife because the custom provides for it. After explaining the story, students will be required to either provide the position of the law on the issues raised in the story or to advise the parties in the story.

The most acceptable way of answering law problem questions is through IRAC method. IRAC is actually the best because it makes it very easy for students to explain any legal principle with authorities in the simplest format. Below are the things you must know about IRAC method of answering law questions.

Meaning of IRAC

The word “IRAC” is an acronym which stands for:

  • Issues – I
  • Rule of law – R
  • Application – A
  • Conclusion – C

Note that if you must answer problem questions using this format, you must have this acronym at the back of your mind. If you miss any step, then you are getting the whole question wrong.

Now, to make sure that you understand the steps listed above, i will take my time to explain what you are supposed to do in every step. Remember, this is to teach you how to answer law problem question using IRAC method.

Your first concern is to determine the issue or issues implicated in the question. This determination involves asking yourself, what is the problem sought to be addressed in the case?

Example: What is the liability of a master regarding tort committed by his servant while engaged in a conduct expressly prohibited by the master?

The importance of accurate identification of the issue(s) is that it narrows your response to the gist of the question.

Once you accomplish this goal, you will know automatically that there is no need to state, for instance, that “the tort borders on vicarious liability” or for you to describe general elements of the tort in question. Your task is to focus only on those elements or information that substantively (not tangentially) speak to the issue(s) you have successfully identified. Relevance is the key here. Recall my admonition, “ the more you write, the more you expose your ignorance .”

Note that you are not expected to call the names of parties in the story in the issues because they are issues for determination in law. You can only mention the names of parties in the story or case given to you when you reach third stage which is APPLICATION

Also read: Are lawyers liars? The truth as to whether lawyers are liars

RULE OF LAW

The “R” or Rule (rule of law) in IRAC is also called “reasoning.” This is because the applicable rule of law is reasoned from the facts of the case. As you think through the problems presented, aided by the issue you have successfully identified, the rule will emerge. What rule of law will guide the court in reaching a correct decision, assuming the same facts?

This is the question that you need to ask your self; it is also the question that you need to address. A rule of law in vicarious liability, for instance, is that “ a master is liable for the acts of his servant, even when expressly prohibited, so long as the servant acted within the scope of his employment. ”

Under the rule of law, students are expected to cite their authorities. Authorities here can be cases, statutes, dictum of judges, articles which are related to the issue in question. It is very important that you cite authorities because that is what will back up the rule of law and legal principles in the case.

APPLICATION:

This is where you apply the rule of law to the issue(s) you have raised. By doing this, you are applying the rule of law to the actual story in the problem question given to you. In the application, you are expected to pick those authorities and rules of law that concerns the issues raised and apply them to the matter effectively.

Always pay attention to exception(s) to the general rule, if any, and clearly outline/justify any distinctions that might be helpful to your argument. By constantly reminding yourself of the issue(s), you are bound to succeed in steering yourself away from irrelevance.

A great analysis is targeted to the issue(s) identified and is judged by the degree of focus/precision as well as the presentation (language/expressions used in articulating your argument).

Finally, the conclusion (“C”) . A few sentences would suffice to wrap up your discussion. Briefly state the outcome of your analysis. Where the question requires that you advise the parties, the conclusion is the best place to do that.

Simply tell each of the parties their rights and persuade them to sue the when the need be. Here, you can also rebuke the party in default in the case and tell him why he/she is at fault.

How to answer law problem question

Okay! Now that you know what all the letters in the acronym IRAC entails, I will give you an example of how a perfect law problem question is answered. The example below is a problem question that borders on the law of defamation.

If you are a student and you don’t know anything about the law of defamation, don’t worry. Just keep reading. You will still be able to understand the steps taken in every section.

Also read: List of less competitive universities in Nigeria 2024

Sample of a law problem question answered using IRAC method

The Daily Trumpeter, a popular Newspaper in Enugu recently published a report of the proceedings of the Enugu State High Court in a land case between Chief Okoto and Barrister Akuepue under the caption ‘Judge calls a popular Enugu Lawyer: ‘A Crook and a Land Speculator”.

In the article, the newspaper reporter, Ade also stated thet Nigerian lawyers are in the habit of using their knowledge of the law to deprive innocent ‘laymen’ of their land. Barrister Akuepue and Barrister Ikpeama, another popular lawyer based in Enugu have sepaprately sued Joe, the edito of The Daily Trumpeter and Ade for publication.

Issue 1: Whether Newspaper Publishers can be liable for publishing court/tribunal proceedings

Issue 2: What must one prove in order for his defense of ‘fair comment’ to be successful

Issue 3: What is the legal position on defamation of a class or a group of person

RULE OF LAW:

Defamation refers to the publication of a statement which is calculated to injure a person and cause right-thinking members of the society to shun or avoid him, or even cause them to hate him and also convey an imputation on him which is injurious to his office, trade or profession – s.137 Enugu State Torts law, cap 150 2004(which shall hereinafter be called ESTL), Sketch v. Ajagbemokeferi.

People are thus warned against idle gossip which may likely impugn another person when communicated to a third party. Defamation may either be in the form of libel which is in a permanent form such as newspaper publication, television or radio broadcasts; or otherwise in slander which has a transient nature usually verbalized or through gesticulations conveying a defamatory connotation. For an action in defamation to succeed, the following essential elements must be proved, as a thing of necessity:

The words complained of must be defamatory:

If right-thinking people of sane minds would think less of an individual or shun and avoid him due to a statement, then this element may be said to have been successfully proved.

Defamatory words must refer to the plaintiff:

It is not sufficient that the defamatory statement described a person merely by his name Akintla v. Anyiam . It is enough where he is identified by his initials, post, Photograph, or even his office – Dafe v. Teswinor.

The words complained of must be published:

It has been held in a vast litany of cases that it is not the publication of defamatory statement but the publication that grounds a cause of action. In fact, in Pullman v. Hill , Lord Esher, Master of the Rolls said thus ‘‘ Publication is the making known of the defamatory matter after it has been written to such person other than the person to whom it is written’’ It therefore follows that publication in itself is what grounds a cause of action- s. 141 ESTL .

Also, communication to the plaintiff himself cannot ground a cause of action for the purposes of determining liability in defamation because defamation is injury to ones reputation and protects not an individual’s opinion of himself but the estimation in which others hold him – Okotcha v. Olumese.

However, even if a person has been alleged to have defamed another, there is an array of defenses open to him. Such defenses are:

  • Unintentional Defamation
  • Innocent dissemination
  • Justification( or truth) – s. 163 ESTL
  • Volenti Non fit injuria – Chapman v.
  • Fair comment- s. 194(1) ESTL

Under the defenses of privilege , we have what is known as absolute privileges and also qualified privileges. Circumstances under which the defense of qualified privilege can arise are varied but for the purposes of our case, we have an occasion known as Statements made in performance of a legal, moral or social duty – s.178 ESTL .

In the connection above, for such an occasion to arise, the person giving out information which is alleged to contain defamatory statements must have a duty to give such information on grounds of public policy and also the party receiving such information, that is, the person to whom the matter is published to must have a corresponding duty of receiving such information.

Perhaps, this is the reason the law admits that radio and television broadcasters as well as Newspaper Publishers and Proprietors are covered by the said defense – NTA v. Babatope . It is also the legal position, pursuant to s.185 (1) that qualified privilege very much applies to those who publish reports of judicial proceedings. However, such broadcasters or publishers must be very careful enough to give reports of what actually took place in court, not necessarily a verbatim report of the proceedings but at least an abridged or condensed report will be privileged, provided that it gives a fair, accurate and correct impression of what transpired.

Also read: Names of Nigerian Presidents From 1960 till Date (Full list) 

It clearly and necessarily follows that when such reports are substantially inaccurate, such a report will lose the protective cloak of this defense. Thus, in Omo-Osagie v, Okutobo, a report of a newspaper of certain court proceedings bore the caption, ‘‘Chief Justice Tells a Teacher: ‘ You are a Bad Woman’.

However, those words were never used by the Judge, and the courts held that such a newspaper report had lost the defense of qualified privilege. The defense of fair comment stated above, consists of criticisms of matters of public interest in the form of comments, by citizens upon true facts, such comments being honestly made without malice – s.194(1) Enugu State Torts law, cap 150 2004.

In order for this defense to avail a person, the following requirements must be proved to the satisfaction of the courts trying the issue:

The Matter commented on must be of Public Interest:

The matter must be one of general concern as to affect the generality of the population at large- London Artists Ltd. v. Littler. Thus issues of land fraud by legal practitioners may fall for issues of public interest.

The Comment must be an expression of Opinion not an assertion of fact:

It is noteworthy that the defense of fair comment consists of two things: a set of facts which must be true and the commentator’s opinion on those facts- s. 194(1) (a). The distinction between a comment and a fact, however, depends on the merits of each case.

The Comment must be Honest- s. 194(1)(c) So long as a commentator honestly expressed his view, it is immaterial that he used excessively strong language o that people read all sorts of innuendoes into it, if he made the comment honestly, he has nothing at all to fear. However, criticisms cannot be used as a cloak for an attack, or for personal imputations on the plaintiff not arising out of the subject matter not based on the facts.

The Comment must be devoid of Malice- s.195 ESTL Malice is a complete bar to a defense of fair comment. Malice means making abuse of the occasion for some indirect purpose – Bakare v. Ibrahim.

When a class or group of persons i.e. lawyers, teachers, doctors etc, are defamed, no individual member of the class is entitled to bring action upon grounds that he has been defamed save only when the class is so small or so ascertainable that what is said of the class is necessarily said of each and every member of it, or if the circumstances of the case show that the plaintiff was singled out and defamed – Knuppfer v. London Express Newspaper Ltd.

Thus, in Zik Enterprises ltd. V. Awolowo , an article which contained defamatory statements against the Action Group was regarded by the court not to refer to the plaintiff but to the Action Group as a political party since it was a large group and the plaintiff could not show circumstances which proved that he was singled out.

Also read: How to answer Law problem questions effectively

APPLICATION

It seems that Daily Trumpeter will not escape liability if the report they gave of the court proceedings was grossly inaccurate or did not give a correct impression of what actually transpired therein. Also, Ade seems to have overstepped the boundaries of fair comment by making such a comment, he seems to have made a statement of fact because saying that someone is in the habit of doing something is as much as imputing a disreputable motive upon him which is very much actionable in law. Hence the cloak of fair comment may no longer protect.

Barrister Akuepue and Barrister Ikpeama by bringing action against Ade seem to be oblivious of the applicable guiding principles in the circumstance. In this connection, a defamatory statement against Nigerian lawyers as a whole is too large a group for the purposes of an action in defamation, unless, if they can show that they were singled out and defamed. They may be going on a wild goose chase.

Joe : You will only escape liability if what you said happened in court was accurate, otherwise, you will fall for liability. Ade: It is true you have overstepped the bounds of fair comment; however you may still escape liability under the principles of defamation of a class or group.

Barrister Akuepue and Barrister Ikpeama : You both can only bring action against Ade and Joe should what Joe reported about the proceedings be inaccurate. With respect to Ade although he has lost the defense of fair comment, he can still be liable for publishing what Joe reported, only the will you have a cause of action against him.

Must read: Most populated universities in Nigeria 2024

4 Things to Note When Answering Law Questions

How to answer law problem question

1. Note the little differences in the way every lecturer want students to answer their questions:

Even though IRAC method of answer law problem questions is generally acceptable, some lecturers make little changes to the way they want their questions to be answered.

For instance, some lecturers posit that after outlining your issues, you must tackle them one after the other. What that means is that, you are expected to apply IRAC in the different issues ( One after the other ). Well, it is impossible for me to cover all the different styles lecturers like. So, I enjoin you to always attend classes. By attending classes, you will know the best method to answer your law problem questions.

It will surprise you to know that some lecturers don’t even like their questions to be answered using IRAC. So you see, you must know what every lecturer wants.

2. Time is a very important factor:

To perform very well as a law student, you must have it in mind that time is very important. You have to be time conscious because you have only but 2:30 minutes to answer all the questions you were given. Most times, students are expected to answer at least 3 questions in law examinations.

So you must learn to manage your time. If you do not manage your time properly, you will definitely not finish answering your questions.

How to answer law problem question

I personally recommend that you get a wristwatch for your exams. It will go a long way to help you plan how to use your time adequately.

3. Always cite your authorities with red pen:

One of the pivotal information that has never been skipped in all the tutorials I have read on how to answer law problem question is the citing of authorities.

The importance of using a red pen when citing authorities in law examinations cannot be over emphasized. The reason is because, it makes it easy for anyone who is assigned to mark your examination script.

So, in other not to jeopardize your chance of success in any law exam, you should use red pen to cite your authorities. Your authorities here may include: cases, names of judges, articles, statutes, dictums etc.

4. Dive detailed information about the issues raised in every case:

Most times, lecturers prefer giving more marks to students who were able to give them detailed information about the question than students who just answered the question. For sure, if you answer the question correctly you will definitely be given the mark you deserve. But if you give more details, you will likely earn more marks.

Take for instance, you might be asked to explain the term “ Nigeria legal system “. If you go ahead to just define it and move to the next question, you won’t even get your full marks there. To get your full marks, you must also highlight the features of Nigerian legal systems and any other subtopic in the topic. Though, you are not expected to go too deep. Just make sure you give detailed information. Some lecturers love it.

Okay! for now, this is all i can disclose on this topic (How to answer law problem question). Like i have rightly mentioned, it is important to always attend classes because the way a particular lecturer will want his/her problem question to be answered might be different from the way another lecturer wants it.

That notwithstanding, i have explained the most generally accepted way of answering law question using IRAC method above. So, if you have not written law examination before, you can safely understand the IRAC method as i explained it. Hope this article was helpful? Do let me know if you have any question or confusion as to how to answer law problem question using IRAC method. I will be glad to help.

answering essay questions in law

Edeh Samuel Chukwuemeka, ACMC, is a lawyer and a certified mediator/conciliator in Nigeria. He is also a developer with knowledge in various programming languages. Samuel is determined to leverage his skills in technology, SEO, and legal practice to revolutionize the legal profession worldwide by creating web and mobile applications that simplify legal research. Sam is also passionate about educating and providing valuable information to people.

Leave a Reply

Save my name, email, and website in this browser for the next time I comment.

  • Faculty and Staff
  • Law School Philosophy
  • The Way From Law Student To Professional Lawyer
  • Student Reviews
  • Bar Review - Repeat Examinee
  • Bar Review - Attorney's Test
  • Bar Boosters
  • Baby Bar Review Workshop
  • Baby Bar Bundle
  • Baby Bar Final Drill - Free Event
  • 1L Survival
  • Exam Solutions - Ace your finals!
  • MBE Review Workshop - Live
  • Multistate Workshop - Science of the Multistate
  • #1 Legal Exam Writing Workshop
  • Live 2-Day Performance Test Workshop
  • Baby Bar Final Drill - 6/15
  • The Art of Test Taking - TBA
  • Private Tutoring | Bar Exam, Baby Bar, and Law School
  • Premier Bar Review & Tutoring
  • Fleming's Fundamentals of Law
  • Nailing the Bar, Tim Tyler Series
  • Seigel Series

Bar Exam Essay Questions With Answers

Passing the bar exam requires more than just knowing the law. Passing the bar exam also requires the ability to properly analyze a bar exam question, organize the issues, and write an effective answer. To demonstrate essay exam proficiency and how to apply the essential test-taking skills, consider the following bar exam sample question below, which will be used as an example to demonstrate how to approach an essay from start to finish, step-by-step, in order to produce a passing written answer.

February 2020 CA Bar Exam Question - Contracts

Barn Exports ("Barn") hired Sam, an up-and-coming artist whose work was recently covered in Modern Buildings Magazine, to paint a one-of-a-kind artistic design along the border of the ceiling in its newly renovated lobby. After discussing the work, Ed, the president of Barn, and Sam signed a mutually drafted handwritten contract, which states in its entirety:

Sam shall paint a unique design along the entire ceiling border of all public areas of the first-floor lobby. Barn shall pay $75,000 upon completion of the work.

When Sam began work, he was surprised that the new plaster ceiling in the lobby had not been sanded and sealed. Sam complained, but was told by Ed that preparation was part of his responsibilities. Although Sam disagreed, he spent four days sanding and sealing the ceiling. When Sam finished painting, he submitted a bill for $78,000, having added $3,000 for labor and supplies used in preparing the ceiling. In response, Barn sent a letter to Sam stating that, because he had not painted the borders in the two public restrooms in the lobby, no payment was yet due. Barn's letter also stated that it had recently spoken to several artists who perform similar work and learned that "surface preparation" was typically the responsibility of the artist.

According to Sam, before the contract was signed, he told Ed that the restrooms could not be included because his paints were not suitable for the high humidity in those locations.

Sam sued Barn for breach of contract in the amount of $78,000.

Barn countersued for specific performance to have the borders in the bathrooms painted.

  • Is Sam likely to prevail in his breach of contract lawsuit against Barn and if so, what damages will he likely recover? Discuss.
  • Is Barn likely to prevail in its lawsuit seeking specific performance against Sam? Discuss.

Step 1 – Call Of The Question

First, always start with the Call of the Question, which is usually located at the end of the exam, because the Call reveals what the Examiners want to know so the facts, yet to be read, can be put in that context. To interpret the Call, use the acronym SPOIL, which stands for the subject matter ( S ), parties ( P ), organizational structure ( O ), issues ( I ), and applicable law ( L ).

Like most Calls, the ones in the Contracts bar exam sample question reveal a wide variety of information, including:

(S) Subjects being tested - Contracts/Remedies:

  • Call 1 specifies a "breach of contract lawsuit" and asks about "damages", which is a remedy in contracts.
  • Call 2 specifies a "lawsuit seeking specific performance." Specific performance is also a remedy in contracts.

(P) Parties: Sam and Barn

  • Call 1 asks about "Sam…in his…lawsuit against Barn."
  • Call 2 asks about "Barn…in its lawsuit…against Sam."

(O) Organization structure to be considered - Sam vs. Barn, followed by Barn vs. Sam

  • Call 1 asks about the "Sam…lawsuit against Barn."
  • Call 2 asks about the Barn…lawsuit…against Sam."

Issues – Breach of contract, damages, and specific performance.

  • Call 1 asks about "breach" and "damages." The second to last line provides "Sam sued Barn" for "$78,000."The facts, yet to be read in their entirety, can be put in the context of whether there was a "breach" by Barn that supports Sam's "$78,000 damages" claim.
  • Call 2 asks about "specific performance." The last line provides "…specific performance to have the borders in the bathrooms painted." The facts, yet to be read in their entirety, can be put in the context of whether the court will find that Sam promised to paint the bathroom borders, and order such performance to Barn.

(L) There is no reference to the applicable law

Contracts test both common law or the Uniform Commercial Code (U.C.C.) The sample exam does not state which body of law to use. Thus, based on the facts, the examinee must make this determination after reading the fact pattern.

2 – Read The Facts To Understand The Story

Second, read the hypothetical facts once for content only. Focus on the story. Pay attention to the names of the parties, dates, quoted language, verbs, adverbs, adjectives, etc. If there is any confusion, read the hypothetical again for clarity.

In reading the exam facts, pay attention to the names of the parties ("Sam" and "Barn") mentioned in the Call. 

"Ed" is not a named party in the Call, but Ed is identified in the first paragraph as "the president of Barn." This reveals that based on Ed's conduct as an agent of Barn, Sam will likely impute arguments against Barn and vice versa.

There is not much in the way of dates, but "surface preparation" is quoted language from Barn's letter.  This is the Examiners' way of flagging that "surface preparation" is going to raise one or more issues and/or arguments. Further, the indentation of the contract terms is equivalent to quoted language, because the Examiners are drawing your attention to that text.

Finally, pay attention to verbs, adverbs, and/or adjectives, which are discussed below in Step 3. If there is any confusion about the content of the Sam v. Barn lawsuit and Barn v. Sam countersuit, read the exam facts again for a full understanding of the story. 

Step 3 – Issue Spot

Third, once understanding the story, begin issue spotting. Start at the beginning of the exam. Go paragraph-by-paragraph, sentence-by-sentence. As each issue is identified, jot it down in the margin of the exam next to the facts that raised it. By starting at the beginning of the fact pattern to issue spot, the first paragraph leads into Ed and Sam signing a contract, which states the terms that are intended for emphasis.

Here, the contract states, "Sam shall paint…" The verb "paint" may raise a goods contract if the contract requires Sam to deliver paint, or a services contract if the contract simply requires Sam to paint an existing area or object.

Further facts that state Sam shall paint "…a unique design along the entire ceiling border…" identify the applicable law as common law because "to paint along a border" is a service.

Continue the issue spotting process as described in the last paragraph of the exam question.

Step 4 – Utilize Mental Checklists And Approaches

Fourth, if during the study process , mental checklists and approaches for issue spotting have been developed, use them to pick up extra issues that could be overlooked without them. Mental checklists and approaches are recommended issue spots tools that every law student should develop during the study process, because they assist issue spotting and the more issues spotted and analyzed in the answer, the higher the grade will be.

In the Contracts bar exam sample question, the fact that Ed and Sam signed the mutually drafted $75,000 contract to paint the design makes offer, acceptance, and consideration fairly straightforward issues that may be picked up without a checklist.

However, defenses are commonly overlooked, which makes a Contracts defense checklist essential to trigger issues that could be missed without one.

The following is an example of the Fleming's Fundamentals Contracts defense approach that illustrates a mnemonic memory device: S o U nless A l I s M istaken, I M issed A erobics F itness D uring P hysical E ducation.  Each capital letter represents a contract defense that may be raised by the facts:

S – Statute of Frauds

U - Unconscionability

A – Adhesion Contract

I - Incapacity

M – Mistake

I – Illegality

M – Misrepresentation

A – Ambiguity

PE – Parol Evidence

In the Contracts sample question, the fact that "According to Sam, before the contract was signed, he told Ed that the restrooms could not be included…" raises the parol evidence rule, which bars oral statements (such as what Sam "told" Ed) made prior to the signed writing. This is critical evidence that Sam needs to introduce and that Barn needs to exclude, because the facts and the Call make Sam's obligation to related to the restrooms an important issue analysis.

Furthermore, one exception to the parol evidence rule is an ambiguity, i.e., allowing extrinsic evidence to resolve whether or not "all public areas of the first-floor lobby" in the contract include or exclude the bathrooms as Barn and Sam assert, respectively.

Step 5 – Reread The Call

Fifth, reread the Call of the Question to confirm a thorough understanding of what the Examiners want in the answer.

  • Call 1, if Sam will likely prevail in Sam v. Barn for breach of contract and if so, what damages will he likely recover. 
  • Call 2, if Barn will likely prevail in Barn v. Sam seeking specific performance.

Step 6 – Organize Issues On An Outline Sheet

Sixth, transfer the issues to a formal outline sheet, making sure they are properly organized. Jot down the formation issues of offer, acceptance and consideration, making sure they are properly organized first under Sam v. Barn (the first lawsuit as Call 1 dictates). Follow with defense issues of parol evidence rule and ambiguity. Continue through the damages issues as requested in the Call.

Repeat the outlining process with its particular issues for Call 2.

Step 7 – Reread Hypothetical One More Time

Seventh, go back and skim the hypothetical one last time to pick up any additional issues that may have been overlooked in steps #3 and #4. Add those issues to the outline.

Step 8 – Double Check Work - Reread The Call One More Time

Eighth, reread the Call of the Question one last time, making sure the issues on the outline are properly organized under the appropriate call if there is more than one.

Step 9 – Write The Final Answer

Finally, write the answer, using the proper presentation for each issue, which is usually referred to as IRAC (issue, rule, analysis, conclusion) or IREAC (issue, rule, explanation, analysis, conclusion).

To check your answer, enter your email address below to download the answer sheet for the above sample bar exam questions. The answer sheet shows you how to use the proper IRAC presentation as demonstrated for each issue above. The answer sheet is a teaching tool to demonstrate the culmination of the above series of sequential steps to follow to properly analyze, organize, and write a passing bar exam essay answer.

Leave a comment

Comments must be approved before appearing

* Required fields

Be in the know and join our mailing list.

Download Your Free Sample Bar Exam Essay Q&A

Added to your cart:.

answering essay questions in law

  • Skip to primary navigation
  • Skip to main content
  • Skip to primary sidebar
  • Skip to footer

Law School Toolbox®

All the tools you need for law school success

A Handy Template for Answering “Hard” Law School Exam Questions

March 22, 2012 By Alison Monahan 7 Comments

OLYMPUS DIGITAL CAMERA

If you think about it dispassionately, this doesn’t make a lot of sense.

Why would you jump right to a conclusion, when you know the analysis is difficult? Um, there’s your answer. If it’s hard to do the analysis, it’s tempting to skip it! (Who really likes doing hard things?)

Why This is a Bad Idea

Writing conclusory answers to hard questions is a bad idea for several reasons:

  • Professors really hate it. Put yourself in the shoes of your professor. You’ve spent a lot of time writing a fiendishly difficult exam, and you’re feeling pretty proud of it. You read the first answer, which is going okay on the warm-up issues, then it gets to the really hard stuff — the stuff you’re particularly pleased with — and says “Clearly Plaintiff has no claim here.” WTF?!? Seriously? You’re going to skip the good stuff and tell me that “clearly” the potential plaintiff has no claim. I hate you! C+. Next!
  • You don’t get very many points for drawing the right conclusions. The point of a law school exam isn’t to get the “right” answer. At best, that gets you maybe 10% of the points. Two students can both get As, and end up with different conclusions on close questions (trust me — happened with two TAs I know). The points are mostly in the analysis. If you want to break it down, it’s probably something like this: issue (10%), rule (10%), conclusion (10%), analysis (70%). If you skip the analysis and jump to the conclusion, you’re leaving A LOT of points on the table.
  • It’s more obvious than you think. There’s something odd about an unsupported conclusion — it really attracts attention. If you’re reading an answer, and someone says “Clearly plaintiff has no claim,” what’s your first reaction? Mine’s to ask “Why?” If there’s no answer to that Why?, it’s very, very obvious that you don’t know what you’re talking about. Not a great impression to leave!

So, the next time you find yourself on an exam writing that something is “obvious” or “clearly” the case, STOP and ask a question: Is this really so clear cut?

If not, deal with the ambiguity , and your grades will improve markedly.

A Template for What to Do Instead

Here’s a template for one way to approach difficult issues. It’s in IRAC format, because lots of professors like that, but you can modify at will.

Steps to go through for a “hard” question:

  • Identify the issue ( The key fact is… )
  • State the rule and how this scenario is different ( Because Plaintiff wasn’t touching the briefcase, Case X doesn’t apply directly. )
  • Analyze each side of the argument ( Defendant will argue… Plaintiff will argue… )
  • Decide who wins AND explain why ( X is likely to win because… )

Pretty simple, right?

Any time you’re feeling panicked, you can fall back on this template. The most important element is #3: “Defendant will argue…” then “Plaintiff will argue….”

If you can make both sides of the argument clearly , with reference to the appropriate legal and factual ambiguities , your conclusion hardly matters. (Yes, you still want to draw a conclusion, and give a good reason after the “because,” but if the question is legitimately difficult, it’s probably a close judgment call about which way the case will come out, so either way is okay.)

— – —

If you found this helpful, why not sign up for our mailing list , and get similar stuff sent right to your inbox! Just ’cause we like you, we’ll also send you our Top 10 exam tips, for free!

Similar posts you might like:

Why Sample Answers are Law School Gold

You’re Totally Unprepared for a Law School Exam! How to Avoid a Disaster

Do You Need to Write Out Practice Exams?

How to Organize Your Exam Answers

Got questions? Leave them in the comments!

Image by mzacha via stock.xchng .

Law School Toolbox logo

Looking for some help to do your best in law school? Find out about our law school tutoring options.

' src=

About Alison Monahan

Alison Monahan is the founder of The Girl's Guide to Law School , which helps law students and prospective law students get in to law school, get through, and stay true to themselves. Alison is a graduate of Columbia Law School, where she was a member of the Columbia Law Review and served as a Civ Pro teaching assistant. You can find her on Twitter at @GirlsGuideToLS .

Reader Interactions

' src=

Legal analysis is more than simply identifying facts that matter on an exam. The writer must demonstrate what it is about the facts or what can be inferred therefrom to show why the rule, element, or standard does or does not apply. Also, the couching of the statement as “A will argue, B will argue” can still be conclusory if the reasoning is not supplied. The fact alone that one side will argue it does not make it any more complete.

Absolutely! However, I would argue the mere act of writing out each side’s arguments will encourage students to think that they might want to do the analysis. In any case, it’s a necessary first step.

' src=

My last semester was very much below my A target. I need guidelines to answer law questions.

Some of these posts might be helpful for you: https://lawschooltoolbox.com/law-school-101/#ExamAnswers . Best of luck!

' src=

These tips were very helpful to me,and i would like to understand more on the IRAC format, and the 10 exam tips.

' src=

Legal writing is bad writing because it is so badly written. Oh wait… sorry, I meant to write, “legal writing is bad” but then had to “law school” the prose to make it sound “fact based.”

Not a big deal… because how absolutely awful legal prose are. See that! The subject came last and I added an unnecessary amplifier for a misplaced adjective- Get it? ha!

One might ask,

‘Why would you jump right to a conclusion, when you know the analysis is difficult?’

One might also write, “The analysis is difficult.”

Of course… no good lawyer would write such a conclusionary (not a real word) statement which one would assume that difficult analysis lends itself to “not jumping to conclusions.”

Yes, this is the common argument made in defense of the horribleness of legal prose: legal prose is somehow different than undergraduate liberal arts arguments because the legal mind must “think differently.” This is really tough analysis so you must not start with a “conclusion.”

While that is “kind of” (but not really) true. You do have to be vague instead of argue a thesis statement- but not true at all because you start by identifying the “issues” and the “rules” that frame your double negatives “but for” the inability to make positive statements (unless asked).

See that, I am writing like a lawyer! I just made a thesis statement while pretending not to make a thesis statement and then digressed into minutia that is stated as a part of a rule.

Ahem! The issues and the rules are not really thesis statements because they are not conclusions.

Never say, “D is liable for hitting P.”

Say, “D is subject to liability for hitting P with a hammer, because D hit P in the head with a hammer, and hitting someone in the head with a hammer is harmful or offensive contact because getting hit in the head with a hammer hurts and/or is offensive.” You say this after stating the rule of course… because god knows, if you don’t repeat parts of the rule in your analysis, then you must not understand the concept.

You must write something like, “C is subject to liability for negligence to J because had C not kicked over the trash can, J would not have fallen down. ‘But for’ the negligence of C, J would not have been hurt by the fall because falling down hurts.” Now, ramble on about actual/ legal cause and add something about proximity/ argue for intervening and lack of intervening causes, for and against proximate causation. Then make the bold conclusion that does not make a conclusion, “C may be subject to liability…”

Congratulations- B plus or better!

See… learn the rules and then write like you are a teenager on LSD explaining something to an alien and you’ll be fine…

Just kidding because I am joking and joking is a form of humor that pokes fun at something because poking fun at something can be funny!

[…] In a nutshell, to break the "too conclusory" habit, force yourself to write down the arguments and counteraguments from each point of view. Yes, it can be tedious but it's what you have to do. Here's a handy template. […]

Leave a Reply Cancel reply

Your email address will not be published. Required fields are marked *

Save my name, email, and website in this browser for the next time I comment.

Want Better Law School Grades?

Sign Up for Our Exam Tips!

  • Privacy Policy
  • Terms & Conditions

Copyright 2024 Law School Toolbox®™

  • My Account |
  • StudentHome |
  • TutorHome |
  • IntranetHome |
  • Contact the OU Contact the OU Contact the OU |
  • Accessibility hub Accessibility hub

Postgraduate

  • International
  • News & media
  • Business & apprenticeships

The Open University Law School

You are here, 6 top tips for answering problem questions in law.

answering essay questions in law

As a law student you’re likely to spend a fair bit of time writing answers to problem questions, so it’s best to be prepared. Below Lawyer Monthly’s latest law school & careers feature benefits from expert top tips from  Emma Jones , Lecturer in Law and member of the  Open Justice  team at The Open University.

Problem questions can help you to develop valuable skills around identifying relevant information, applying legal principles to specific scenarios and writing advice in a clear and logical manner. Here are some top tips on how to approach this type of question.

1. Read the question carefully

OK, so this really applies to all types of assignments, but with problem questions there can be a pretty lengthy scenario for you to get to grips with. It can help to highlight or underline, but even better try making a flow chart or chronology of events, or a spider diagram detailing the involvement of each party.

2. Find a way to break down the question.

One common way to approach analysing problem questions is the IRAC method – identify the Issue, explain the Legal Rule, set out its Application and reach a Conclusion based on this. Depending on the scenario you’re given, you might need to work through this process several times, for example, once for each party involved or each potential cause of action

3. Show what you know

When it comes to explaining the legal rules that apply to a scenario, it can be tempting to quote sections of statute or parts of judgments. Although it’s great to reference legislation and cases, setting out their meaning in your own words really demonstrates your understanding. It can be tricky to get the balance between keeping the original meaning and putting it in your own way, but it does get easier with practice.

4. Reason, reason, reason!

The Application part of a problem question is key. It can be very tempting to jump from the legal rule to a conclusion, but you need to take your reader through your thought-process step-by-step. Often, there is no one “right” answer to a scenario, the key is to construct a clear and sound argument using legal authorities and explaining how they apply to the facts.

5. Get the structure and presentation right

This leads neatly onto the next point – structuring your work carefully. Your Law School may have its own rules on this, for example, whether or not to include a brief introduction and when to use headings. It is important to follow these. The general rule is to try and make your structure and writing as easy to follow as possible. Imagine you are writing for an intelligent lay person with no previous knowledge of law. In fact, you can always ask a friend or family member to take a look to see if they can follow what you’re saying.

6. Reaching a conclusion

When you are trying to write a conclusion, you may find that there are parts of the scenario that are a little ambiguous or where there is potential for different outcomes. If that is the case, it is fine to indicate that you can’t reach a final conclusion, but it is important to explain why not. On the other hand, if you can give a conclusion, you should try and do so. It’s usually fairly clear when someone has lacked the confidence to make a decision.

Problem questions can be challenging, but they are a great way of developing key skills which are needed in plenty of careers, not least for working on the legal profession. Just remember, one day you may have a real client in front of you, and be very glad you had the chance to practice first!

This article was originally published in  Lawyer Monthly . Click to read the  original article .

Get in touch

Contact the Law School

The Open University

  • Study with us
  • Work with us
  • Supported distance learning
  • Funding your studies
  • International students
  • Global reputation
  • Sustainability
  • Apprenticeships
  • Develop your workforce
  • Contact the OU

Undergraduate

  • Arts and Humanities
  • Art History
  • Business and Management
  • Combined Studies
  • Computing and IT
  • Counselling
  • Creative Arts
  • Creative Writing
  • Criminology
  • Early Years
  • Electronic Engineering
  • Engineering
  • Environment
  • Film and Media
  • Health and Social Care
  • Health and Wellbeing
  • Health Sciences
  • International Studies
  • Mathematics
  • Mental Health
  • Nursing and Healthcare
  • Religious Studies
  • Social Sciences
  • Social Work
  • Software Engineering
  • Sport and Fitness
  • Postgraduate study
  • Research degrees
  • Masters in Social Work (MA)
  • Masters in Economics (MSc)
  • Masters in Creative Writing (MA)
  • Masters in Education (MA/MEd)
  • Masters in Engineering (MSc)
  • Masters in English Literature (MA)
  • Masters in History (MA)
  • Masters in International Relations (MA)
  • Masters in Finance (MSc)
  • Masters in Cyber Security (MSc)
  • Masters in Psychology (MSc)
  • A to Z of Masters degrees
  • OU Accessibility statement
  • Conditions of use
  • Privacy policy
  • Cookie policy
  • Manage cookie preferences
  • Modern slavery act (pdf 149kb)

Follow us on Social media

Google+

  • Student Policies and Regulations
  • Student Charter
  • System Status
  • Contact the OU Contact the OU
  • Modern Slavery Act (pdf 149kb)

© . . .

  • White House
  • Energy/Environment
  • Health Care
  • Transportation
  • Heard on the Hill
  • Fintech Beat
  • Political Theater
  • Donald Trump
  • White House Calendar
  • White House Releases
  • Press Seating Chart
  • Donald Trump Twitter
  • Correspondents Dinner
  • Newsletters
  • Capitol Ink
  • Roll Call e-Edition
  • Classifieds

Biden gave debate performance Democrats feared, but Trump did not win new votes

Ahead of the debate, the Trump team released an ad they planned to run after the event arguing that a vote for Biden is really a vote for Vice President Kamala Harris, seemingly predicting Biden’s performance.

“You know who’s waiting behind him, right?” the ad’s narrator said.

“Listen, people can debate on style points, but ultimately this election and who is the president of the United States has to be about substance, and the contrast is clear,” Harris said on CNN after the debate, trying to turn the focus to Trump. “He would not disavow what happened on Jan. 6. He would not give a clear answer on whether he would stand by the election results this November.”

Harris also referenced the Trump-appointed Supreme Court justices helping to overturn Roe v. Wade — but Trump himself seemed to waffle on a particular position on abortion, arguing repeatedly that the issue was in the hands of states now and falsely saying it was a universal goal he’d delivered.

[ Fact-checking the Biden-Trump debate ]

“Everybody wanted to get it back to the states,” Trump said. “Everybody without exception — Democrats, Republicans, liberals, conservatives — everybody wanted it back.”

Biden landed jabs, repeatedly arguing that Trump was not telling the truth, that historians had voted Trump the worst president in history, and calling him a “whiner” who could not accept his loss in 2020 and would refuse to do so again this year.

“The idea if you lose again, you accepting anything — you can’t stand the loss,” Biden said. “Something snapped in you when you lost the last time.”

There was also this historic exchange, after Trump called Biden “a criminal” for demanding Ukraine replace a corrupt prosecutor:

“The idea that I did anything wrong relative to what you’re talking about is outrageous. It’s simply a lie, No. 1,” Biden said. “No. 2, the idea that you have a right to seek retribution against any American just because you’re president is wrong. It’s simply wrong. No president has ever spoken like that before. No president in our history has spoken like that before. No. 3, the crimes you are still charged with. And think of all the civil penalties you have. How many billions of dollars do you owe in civil penalties? Or for molesting a woman in public, for doing a whole range of things, of having sex with a porn star on the night while your wife was pregnant. I mean, what what what are you talking about? You have the morals of an alley cat.”

“I didn’t have sex with a porn star,” Trump replied, accusing Biden of using the Justice Department to go after him — even though Trump was convicted by the Manhattan district attorney in New York City. “Because he thought it was going to damage me. But when the public found out about these cases, because they understand it better than he does, he has no idea what these cases are. But when he — when they found out about these cases, you know what they did? My poll numbers went up. Way up.”

But Biden also appeared to freeze at one point in an answer before saying, “We finally beat Medicare.” And in one answer about Ukraine, Biden used Trump’s name when he appeared to be referring to Russian President Vladimir Putin.

Afterward, his allies focused statements largely on policy, and some did not even try to sugarcoat Biden’s performance.

Former Sen. Claire McCaskill, D-Mo., said on MSNBC that she wished she were in the role of debate surrogate so she could look to the positive, but in the role of network commentator she spoke directly about how poorly she thought Biden had performed.

“My job now is to be really honest. Joe Biden had one thing he had to do tonight, and he didn’t do it. He had one thing he had to accomplish, and that was to reassure America that he was up to the job at his age,” McCaskill said. “And he failed at that tonight.”

McCaskill said she was hearing from elected officials, including those in offices where “you might know where they serve.”

“I don’t know if things can be done to fix this,” she said, noting that Harris and California Gov. Gavin Newsom were effective surrogates Thursday night.

“Those two people are signaling to a whole lot of Americans that are paying attention, how come they’re not running? How come the Democratic Party doesn’t have them at the top of the ticket instead of using them to shore up what have become after tonight some pretty glaring weaknesses … in our president,” McCaskill said.

Newsom was in Georgia as one of the top surrogates in the spin room for the Biden-Harris campaign, appearing along with Sen. Raphael Warnock, D-Ga.

As with the vice president and Newsom, Warnock and other surrogates speaking on Biden’s behalf argued for the election continuing to be about issues and policies.

“I would be concerned if the president didn’t have a record to run on, but the fact of the matter is that this is a man who has passed historic legislation,” Warnock told reporters after the debate.

Democrats also tried to keep the focus on Trump.

Rep. Jasmine Crockett, D-Texas, said Trump “was acting as if he was some used car salesman and could just tell us whatever and pretend as if it was fact.”

The question may be less whether Biden’s showing Thursday will drive voters to Trump, however, or whether it will boost voter apathy, driving them stay home, to cast a ballot for Robert F. Kennedy Jr., or — as is possible in Nevada — to vote for none of the candidates whose name appear on the ballot.

Recent Stories

Sen. Mike Lee says the high court’s decision represents the first step for Congress to make more decisions when writing laws.

Supreme Court’s ‘Chevron’ ruling means changes for writing laws

Sen. Bob Menendez, D-N.J., arrives for trial June 11 at federal court in New York City.

Prosecution rests in Sen. Bob Menendez corruption trial

Steve Bannon, with his attorney David Schoen, left, is seen after being found guilty of contempt of Congress in 2022.

Supreme Court denies Steve Bannon bid to remain out of prison

President Joe Biden, right, and former President and Republican presidential candidate Donald Trump participate in the presidential debate at the CNN Studios in Atlanta, Georgia, on June 27, 2024.

Debate debacle — Congressional Hits and Misses

President Joe Biden looks down as he participates in the first presidential debate of the 2024 election cycle Thursday in Atlanta.

Democrats uneasy about Biden’s ‘crappy’ debate night

Homeless rights activists rally outside of the Supreme Court on April 22, the day of oral argument in City of Grants Pass, Oregon v. Johnson.

Supreme Court upholds Oregon city law targeting homeless

Fact-checking Biden and Trump's claims at the first debate

Forget alternative facts and political spin: Thursday's presidential debate was more like a tsunami of falsity.

Former President Donald Trump unleashed a torrent of misinformation on topics from terrorism to taxes during the first debate of the 2024 general election, while President Joe Biden flubbed figures and facts about military deaths and insulin prices.

More than a dozen NBC News reporters, editors and correspondents fact-checked the key claims the presidential candidates made Thursday night. Here they are by topic:

Economy, trade and health care

Fact check: did biden inherit 9% inflation.

“He also said he inherited 9% inflation. Now, he inherited almost no inflation, and it stayed that way for 14 months, and then it blew up under his leadership,” Trump said about Biden.

This is false.

The inflation rate when Biden took office in January 2021 wasn’t 9%. It was 1.4%. It has risen on his watch, peaking at about 9.1% in June 2022, but by last month it had come down to 3.3%. Pandemic-related stimulus policies put in place by both Trump and Biden were blamed, in part, for the rise in the inflation rate.

Fact check: Did Biden lower the cost of insulin to $15 a shot?

“We brought down the price of prescription drugs, which is a major issue for many people, to $15 for an insulin shot — as opposed to $400,” Biden said.

Biden capped the cost of insulin at $35 a month under Medicare, not $15 a shot, and some drug companies have matched that cap. The price cap doesn’t apply to everyone , however. 

What’s more, Biden’s also significantly overstating how much insulin cost before the change. A 2022 report by the Department of Health and Human Services found that patients using insulin spent an average of $434 annually on insulin in 2019 — not $400 a shot.

Fact check: Did Trump lower the cost of insulin?

Trump claimed credit for lowering the cost of insulin for seniors, saying, “I am the one who got the insulin down for the seniors.”

That is mostly false.

In 2020, Trump created a voluntary program under Medicare Part D. The program allowed Medicare Part D plans to offer some insulin products for no more than $35 per month. It was active from 2021 to 2023, with fewer than half of the plans participating each year. 

In 2022, Biden signed the Inflation Reduction Act, which included a provision that lowered the out-of-pocket cost for people on Medicare to $35 a month and covered all insulin products. The cap didn’t apply to those with private insurance. However, after the law was implemented, insulin manufacturers voluntarily lowered the out-of-pocket cost to $35 a month for people with private insurance.

Fact check: Does Biden want to raise ‘everybody’s taxes’ by four times?

“Nobody ever cut taxes like us. He wants to raise your taxes by four times. He wants to raise everybody’s taxes by four times,” Trump claimed. “He wants the Trump tax cuts to expire.”

Biden’s tax plan “holds harmless for 98% of households,” said Kyle Pomerleau, senior fellow at the conservative American Enterprise Institute. And Biden wants to extend the majority of the Trump tax cuts, too, though he has advocated for hiking taxes on very high earners.

Fact check: Biden said the U.S. trade deficit with China is at its lowest since 2010

“We are at the lowest trade deficit with China since 2010,” Biden said.

This is true.

The U.S. had $279 billion more in imports than exports to China last year, the lowest trade deficit with the world’s second-largest economy since 2010. The highest deficit in recent years was $418 billion, in 2018, when Trump began a trade war with China. 

The decline has been driven largely by tariffs that Trump imposed in office and that Biden has maintained and in some cases expanded.

Fact check: Are immigrants taking ‘Black jobs’?

Asked about Black voters who are disappointed with their economic progress, Trump claimed Black Americans are losing their jobs because of illegal border crossings under Biden’s administration.

“The fact is that his big kill on the Black people is the millions of people that he’s allowed to come through the border. They’re taking Black jobs now,” Trump said.

There’s no evidence that undocumented immigrants are taking jobs away from Black Americans. In fact, according  to the Bureau of Labor Statistics , the Black unemployment rate fell to 4.8% in April 2023 — an all-time low. Before that, the Black unemployment rate was as high as 10.2% in April 2021.

Immigration

Fact check: did trump end catch and release.

“We ended ‘catch and release,’” Trump said.

Trump did not end “catch and release,” a term used to describe the practice of releasing migrants into the country with court dates while they await court hearings. The U.S. doesn’t have enough facilities to detain every migrant who crosses the border until they can see judges, no matter who is president, so Trump — like Barack Obama before him and Biden after him — released many migrants back into the U.S.

Fact check: Did the Border Patrol union endorse Biden?

“By the way, the Border Patrol endorsed me, endorsed my position,” Biden said.

The National Border Patrol Council, the labor union for U.S. Border Patrol agents and staff members, has endorsed Trump. 

“The National Border Patrol Council has proudly endorsed Donald J. Trump for President of the United States,” the group’s vice president, Hector Garza, said in a statement shared exclusively with NBC News. 

The union posted on X , “to be clear, we never have and never will endorse Biden.”

Biden may have been referring to a Senate immigration bill that he backed, which earned the union’s endorsement .

Fact check: Did Trump have ‘the safest border in the history of our country’?

“We had the safest border in the history of our country,” Trump said.

It’s a clear exaggeration. In 2019, the last year before the Covid-19 pandemic brought down border crossings, there were roughly 860,000 illegal border crossings, far more than in any year during the Obama administration.

Fact check: Trump says Biden is allowing ‘millions’ of criminals to enter U.S.

“I’d love to ask him … why he’s allowed millions of people to come in from prisons, jails and mental institutions to come into our country and destroy our country,” Trump said.

There is no evidence of this.

Venezuela doesn’t share law enforcement information with U.S. authorities, making it very hard to verify criminal histories of immigrants coming to the U.S. But there’s no evidence that Venezuela is purposefully sending “millions” of people from mental institutions and prisons to the U.S.

Fact check: Did Virginia’s former governor support infanticide?

“They will take the life of a child in the eighth month, the ninth month and even after birth. After birth. If you look at the former governor of Virginia, he was willing to do so, and we’ll determine what we do with the baby. Meaning we’ll kill the baby. ... So that means he can take the life of the baby in the ninth month and even after birth. Because some states, Democrat-run, take it after birth. Again, the governor, the former Virginia governor, put the baby down so that we decide what to do with it. He’s willing to, as we say, rip the baby out of the womb in the ninth month and kill the baby. Nobody wants that to happen, Democrat or Republican; nobody wants it to happen,” Trump said.

While some Democrats support broad access to abortion regardless of gestation age, infanticide is illegal, and no Democrats advocate for it. Just 1% of abortions are performed after 21 weeks’ gestation, according to the Centers for Disease Control and Prevention .

Trump first made the claim in 2019, after Virginia’s governor at the time, Ralph Northam, made controversial remarks in discussing an abortion bill. NBC News debunked the claim then, reporting that Northam’s remarks were about resuscitating infants with severe deformities or nonviable pregnancies. 

Asked on a radio program what happens when a woman who is going into labor desires a third-trimester abortion, Northam noted that such procedures occur only in cases of severe deformities or nonviable pregnancies. He said that in those scenarios, “the infant would be resuscitated if that’s what the mother and the family desired, and then a discussion would ensue between the physicians and the mother.”

Terrorism, foreign policy and the military

Fact check: trump said there was ‘no terror’ during his tenure.

“That’s why you had no terror, at all, during my administration. This place, the whole world, is blowing up under him,” Trump said.

There were two ISIS-inspired terrorist attacks while Trump was president. The first occurred in October 2017, when Sayfullo Saipov killed eight people and injured a dozen more in a vehicle ramming attack on the West Side Highway bike path in New York City. The second occurred in December 2017, when Akayed Ullah injured four people when he set off a bomb strapped to himself.

Fact check: Biden suggests no troops died under his watch

“The truth is I’m the only president this century that doesn’t have any this decade and any troops dying anywhere in the world like he did,” Biden said.

The Defense Department confirmed that 13 U.S. service members were killed in a suicide bombing attack at Abbey Gate at the Kabul airport by a member of ISIS-K as the U.S. was leaving Afghanistan. 

Environment

Fact check: did trump have the ‘best environmental numbers ever’.

“During my four years, I had the best environmental numbers ever, and my top environmental people gave me that statistic just before I walked on the stage, actually,” Trump said.

The figure Trump is referring to is the fact that carbon emissions fell during his administration. He posted the talking points his former Environmental Protection Agency chief emailed him on social media before the debate.

And it’s true that carbon emissions are falling — they have been dropping for years. Emissions particularly plunged in 2020, dropping to levels around those in 1983 and 1984. That drop was in large part thanks to Covid lockdowns, and emissions rose again when air travel and in-person working resumed. 

Still, climate activists and experts are quick to note that those drops are nowhere near enough to head off predicted catastrophic effects of global warming. Other major countries cut their emissions at a much faster rate during the Trump administration.

Fact check: The Jan. 6 crowd was not ‘ushered in’ by the police

“If you would see my statements that I made on Twitter at the time and also my statement that I made in the Rose Garden, you would say it’s one of the strongest statements you’ve ever seen. In addition to the speech I made in front of, I believe, the largest crowd I’ve ever spoken to, and I will tell you, nobody ever talks about that. They talk about a relatively small number of people that went to the Capitol and, in many cases, were ushered in by the police. And as Nancy Pelosi said, it was her responsibility, not mine. She said that loud and clear,” Trump said.

During a lengthy answer to a question about whether he would accept the result of the 2024 election and say all political violence is unacceptable, Trump made several false statements, including the claim that police “ushered” rioters into the U.S. Capitol and that then-House Speaker Nancy Pelosi, D-Calif., said it was her responsibility to keep the chamber safe. 

Video and news reports of the Jan. 6 riots clearly captured the U.S. Capitol under attack by pro-Trump crowds who overran the law enforcement presence around and inside the complex. 

On Pelosi, Trump was most likely referring to video shot by Pelosi’s daughter Alexandra for an HBO documentary that showed her during the events of Jan. 6, 2021, tensely wondering how the Capitol was allowed to be stormed.

“We have responsibility, Terri,” Pelosi tells her chief of staff, Terri McCullough, as they leave the Capitol in a vehicle. “We did not have any accountability for what was going on there, and we should have. This is ridiculous.”

“You’re going to ask me in the middle of the thing, when they’ve already breached the inaugural stuff, ‘Should we call the Capitol Police?’ I mean the National Guard. Why weren’t the National Guard there to begin with?” Pelosi says in the video. 

“They clearly didn’t know, and I take responsibility for not having them just prepare for more,” she says. 

Many allies of Trump have tried for the more than three years since the riots to paint Pelosi as somehow being responsible for the violence. Some Trump-backing Republicans have, for example, falsely claimed that she blocked the National Guard from going to the Capitol during the riots.

And everything else ...

Fact check: trump skipped world war i cemetery visit because the soldiers who died were ‘losers’.

Biden said that Trump “refused to go to” a World War I cemetery and that “he was standing with his four-star general” who said Trump said, “I don’t want to go in there, because they’re a bunch of losers and suckers.”

In 2018, during a trip to France, Trump canceled a visit to an American cemetery near Paris, blaming weather for the decision. 

But in September 2020, The Atlantic reported that Trump had axed the visit because he felt that those who’d lost their lives and been buried there were “losers.” The magazine cited “four people with firsthand knowledge of those discussions.”

According to The Atlantic, Trump said: “Why should I go to that cemetery? It’s filled with losers.” In another conversation, The Atlantic reported, Trump said the 1,800 American Marines who died were “suckers.” 

Several media outlets confirmed the remarks, and Trump’s former White House chief of staff John Kelly also said those specific comments were true.

Fact check: Trump says Biden didn’t run for president due to 2017 Charlottesville rally

“He made up the Charlottesville story, and you’ll see it’s debunked all over the place. Every anchor has — every reasonable anchor has debunked it, and just the other day it came out where it was fully debunked. It’s a nonsense story. He knows that, and he didn’t run because of Charlottesville. He used that as an excuse to run,” Trump said about Biden.

The “Unite the Right” rally in Charlottesville, Virginia, in 2017 featured torch-bearing white supremacists marching to protest the removal of a Robert E. Lee statue and chanting racist slogans like “You will not replace us.” It turned deadly when a car plowed into a crowd .

In recent months, Trump has downplayed the violence, saying it was “nothing” compared to recent pro-Palestinian protests on university campuses.

Meanwhile, Biden has always pointed to Trump’s 2017 comments as the primary reason he decided to seek the presidency in 2020, including in his campaign announcement video back in April 2019 .

answering essay questions in law

Jane C. Timm is a senior reporter for NBC News.

answering essay questions in law

Julia Ainsley is the homeland security correspondent for NBC News and covers the Department of Homeland Security for the NBC News Investigative Unit.

answering essay questions in law

Adam Edelman is a political reporter for NBC News.

answering essay questions in law

Tom Winter is a New York-based correspondent covering crime, courts, terrorism and financial fraud on the East Coast for the NBC News Investigative Unit.

MBA Watch Logo

Revealed: Harvard Business School’s New MBA Essays For Applicants

  • Share on Facebook
  • Share on Twitter
  • Share on LinkedIn
  • Share on WhatsApp
  • Share on Reddit

answering essay questions in law

Harvard Business School’s Baker Library.

With just 10 weeks before its first application deadline on Sept. 4th, Harvard Business School today (June 25) revealed a newly revised application for MBA candidates, including a new set of three short essays along with a refresh on how it will evaluate applicants for future classes.

The new prompts?

Business-Minded Essay : Please reflect on how your experiences have influenced your career choices and aspirations and the impact you will have on the businesses, organizations, and communities you plan to serve. (up to 300 words)

Leadership-Focused Essay : What experiences have shaped who you are, how you invest in others, and what kind of leader you want to become? (up to 250 words)

Growth-Oriented Essay : Curiosity can be seen in many ways. Please share an example of how you have demonstrated curiosity and how that has influenced your growth. (up to 250 words)

NEW HARVARD BUSINESS SCHOOL ESSAYS PUT THROUGH BY NEW MBA ADMISSIONS CHIEF

Eagerly awaited by thousands of prospective students and admission consultants, you can bet that the admissions pages of the HBS website were continually refreshed all morning for a glimpse at the new essay. The Harvard Business School essay prompt for the Class of 2027 was posted at 10:30 a.m. with the opening of the 2024-2025 application online.

This year’s change was put through by Rupal Gadhia , who joined the school as managing director of admissions and financial aid last October. A 2004 Harvard MBA, Gadhia came to the school with no previous admissions experience, having been the global head of marketing for SharkNinja robots.

In explaining the change in a blog post , Gadhia noted that “we have refreshed the criteria on which we evaluate candidates. We are looking for applicants who are business-minded, leadership-focused, and growth-oriented…This is your opportunity to discuss meaningful or formative experiences that are important to you that you haven’t had a chance to fully explore elsewhere in your application…Be authentic, be yourself.”

WHAT HARVARD BUSINESS SCHOOL IS REALLY LOOKING FOR IN THE NEW ESSAYS

The school added some context to its new criteria for admission, more clearly defining what it means by business-minded, leadership-focused, and growth-oriented.

Business-Minded

We are looking for individuals who are passionate about using business as a force for good – who strive to improve and transform companies, industries, and the world. We are seeking those who are eager to solve today’s biggest problems and shape the future through creative and integrated thinking. Being business-minded is about the interest to help organizations succeed, whether in the private, public, or non-profit sector. This business inclination can be found in individuals with a variety of professional and educational experiences, not just those who come from traditional business backgrounds.

In Your Application: We will look for evidence of your interpersonal skills, quantitative abilities, and the ways in which you plan to create impact through business in the future.

Leadership-Focused

We are looking for individuals who aspire to lead others toward making a difference in the world, and those who recognize that to build and sustain successful organizations, they must develop and nurture diverse teams. Leadership takes many forms in many contexts – you do not have to have a formal leadership role to make a difference. We deliberately create a class that includes different kinds of leaders, from the front-line manager to the startup founder to the behind-the-scenes thought leader.

In Your Application: Your leadership impact may be most evident in extracurriculars, community initiatives, or your professional work.

Growth-Oriented

We are looking for individuals who desire to broaden their perspectives through creative problem solving, active listening, and lively discussion. At HBS you will be surrounded by future leaders from around the world who will make you think more expansively about what impact you might have. Our case and field-based learning methods depend on the active participation of curious students who are excited to listen and learn from faculty and classmates, as well as contribute their own ideas and perspectives.

In Your Application: We will look for the ways in which you have grown, developed, and how you engage with the world around you.

TIGHTER TIMEFRAME FOR ROUND ONE APPLICANTS

The new essay prompts come  nearly two months after candidates to the school’s MBA program would more typically know what was expected of them. Some admission consultants say the delay over the prompt’s release, along with nearly a month’s slow down in releasing application deadlines, is “wildly insensitive” to applicants who will have less time than normal to prepare for the round one deadline of Sept. 4th.

That’s especially true because the most successful applicants to HBS have highly demanding jobs that consume the vast majority of their time. Many candidates go through multiple drafts of their essays to get them as close to perfection as humanly possible. MBA admission consultants are expecting a lot of up-to-the-deadline work this year to help prep candidates for Harvard and other top business schools.

The new application still preserves the post-interview reflection for applicants who are invited to a 30-minute admissions interview. Within 24 hours of the interview, candidates are required to submit a written reflection through the school’s online application system.

REACTION TO THE NEW CHANGE IS MIXED

Early reaction to the change suggests the likelihood of mixed reviews. “This is an uninspired and odd set of questions,” says Sandy Kreisberg, founder of HBSGuru.com and an MBA admissions consultant who closely reads the tea leaves of Harvard’s admissions process. “I don’t know how it’s different from what else do you want us to know about you, frankly,” he adds in a reference to last year’s single essay prompt.

“HBS has certainly moved from the abstract to the concrete,” believes Jeremy Shinewald, founder and CEO of mbaMission, a leading MBA admissions consulting firm. “Some applicants previously felt like they didn’t know where to start and some weren’t sure if they had answered the question, even when they were done. Now, the questions are quite straightforward and all have a cause and effect relationship — one where the applicant discusses the past to reveal the present or future. Smart applicants will understand how to share their experiences and, more importantly, how to relay their values. Some will mistakenly try to whack HBS over the head with stories of their epic feats, but the key isn’t to brag or embellish – the key is to simply create a clear relationship, via narrative, between past experience and true motivations.”

Shinewald found it astonishing that Harvard could not have made the change earlier. “It is, of course, surprising that HBS left applicants on edge until the last minute, all to create very traditional essays,” he adds. “As applicants learn in MBA classrooms, change can be hard and take time. The bottom line here is that these essays are somewhat of an applicant’s dream – they allow the savvy applicant to play to their strengths and draw on their best anecdotes and experiences to create a complete story. Some applicants will lament the absence of a ‘Why HBS?’ prompt, but my guess is that the admissions committee recognized that they would get an almost homogenous collection of essays touting the case method and other well known features. HBS gets some kudos for keeping the focus on the applicant.”

Adds Petia Whitmore of My MBA Path: “I think they reflect one of the traits of this new generation of candidates which is that they don’t handle ambiguity well. So it seems like Harvard had to spell out what they’re looking for way more prescriptively than in the past.”

Some, however, find the new essays a return to the past. “To me, the prompts feel quite regressive, and a return to the more formulaic approach that pervaded MBA applications two decades ago,” believes Justin Marshall, a New York-based MBA admissions consultant. “Because the previous prompt was so open ended, it forced applicants to be introspective and self-aware. You couldn’t just ramble for 900 words; you had to identify themes in your life to show how your personal experiences shaped your values, your leadership style, and your goals. Comparatively, these new prompts are much more paint-by-numbers. Applicants will likely cover the same ground in terms of topic, but there’s very little room for nuance and self-expression. I think it will be harder for applicants with less conventional backgrounds and experiences to differentiate themselves. I’m sure HBS grew tired of reading so many painfully earnest ‘life story’ essays, but I suspect they’ll soon find themselves yearning for essays that have a heartbeat and personality. 250 words just doesn’t allow for that unless you’re a very crafty writer.”

Whatever the case, getting into Harvard’s MBA program is still a daunting exercise. Last year, 1,076 of the 8,264 candidates who applied for admission to Harvard Business School gained admission, an acceptance rate of 13.2%, making HBS the second most selective prestige MBA program in the country after Stanford Graduate School of Business which had an admit rate of 8.4%. Harvard saw a 15.4% drop in MBA applications from the 9,773 it received a year-earlier.

Joint degree applicants for the Harvard Medical School, Harvard School of Dental Medicine, Harvard Graduate School of Arts and Sciences, Harvard Law School, and Harvard Kennedy School must provide an additional essay: How do you expect the joint degree experience to benefit you on both a professional and a personal level? (up to 400 words)

BIGGEST CHANGE IN HARVARD BUSINESS SCHOOL ESSAY IN NEARLY A DECADE

Joint degree applicants for the Harvard Paulson School of Engineering and Applied Sciences must provide an additional essay: The MS/MBA Engineering Sciences program is focused on entrepreneurship, design, and innovation. Describe your past experiences in these areas and your reasons for pursuing a program with this focus. (recommended length: 500 words). Applicants will also be able to respond to an optional essay.

In any case, it’s the biggest change in Harvard Business School’s application in nearly a decade. The last time HBS made a major switch, moving to the essay prompt it just eliminated, was in 2016. That change to just one essay with no word limit and a post-interview reflection was made by then admissions chief Dee Leopold.

When Leopold applied to Harvard as an MBA candidate in 1978, she had to write eight essays. Over her years as managing director of admissions, she first cut the essays down to four and then one, making it optional, and finally the one last prompt with a post-interview reflection, saying that applying to HBS should not be a writing contest .

answering essay questions in law

OUR BUSINESS CASUAL PODCAST: The New HARVARD BUSINESS SCHOOL MBA Application:   Fortuna Admissions’ Caroline Diarte-Edwards and ApplicantLab’s Maria Wich-Vila join P&Q’s John A. Byrne to offer applicant advice on how to answer the new HBS essay prompts

DON’T MISS: 2024-2024 MBA APPLICATION DEADLINES or  HARVARD BUSINESS SCHOOL WILL NOW UPDATE ITS MBA ESSAY 

Questions about this article? Email us or leave a comment below.

  • Stay Informed. Sign Up! Login Logout Search for:

answering essay questions in law

What Harvard Business School Really Wants: How To Ace The HBS Essay

answering essay questions in law

How To Ace The INSEAD Video Questions

answering essay questions in law

Why Are MBA Application Goals Important?

Karen Marks, president and founder of North Star Admissions Consulting

How Can I Improve My Chances Of Getting Into Business School – With A Scholarship?

  • How To Use Poets&Quants MBA Admissions Consultant Directory
  • How To Select An MBA Admissions Consultant
  • MBA Admission Consulting Claims: How Credible?
  • Suddenly Cozy: MBA Consultants and B-Schools
  • The Cost: $6,850 Result: B-School

Our Partner Sites: Poets&Quants for Execs | Poets&Quants for Undergrads | Tipping the Scales | We See Genius

  • Find a Lawyer
  • Ask a Lawyer
  • Research the Law
  • Law Schools
  • Laws & Regs
  • Newsletters
  • Justia Connect
  • Pro Membership
  • Basic Membership
  • Justia Lawyer Directory
  • Platinum Placements
  • Gold Placements
  • Justia Elevate
  • Justia Amplify
  • PPC Management
  • Google Business Profile
  • Social Media
  • Justia Onward Blog

Q: I have filed a Protection From Stalking order in the State of Kansas and a temporary order has been granted.

The judge required that the defendant Cease and Desist mentioning me in any manner on social media posts, including YT. The defendant filed a motion to dismiss, stating that she has freedom of speech and can say whatever she wants. I need to file a response, but I have no idea how to respond. I am not a public figure, the statements she has made have been dangerous, alarming and many qualify as defaming/false light. it has been a severe invasion of privacy, and she ignored a Cease and Desist from an attorney.

James L. Arrasmith

  • Civil Rights Lawyer
  • Sacramento, CA
  • (916) 704-3009
  • Email Lawyer
  • View Website

A: Your situation sounds challenging and concerning. Here's some information that may help you formulate a response: 1. Freedom of speech is not absolute: While the First Amendment protects free speech, there are limitations, especially when it comes to harassment, stalking, and defamation. 2. Court orders supersede general free speech rights: When a judge issues an order, it becomes legally binding. The defendant must comply with the order regardless of their personal beliefs about free speech. 3. Private citizens have stronger privacy protections: Since you're not a public figure, you have greater legal protections against defamation and invasion of privacy. 4. Defamation and false light: If the statements made about you are false and damaging to your reputation, they may not be protected speech. 5. Prior attempts to stop the behavior: The fact that the defendant ignored a cease and desist letter from an attorney demonstrates a pattern of disregard for your privacy and legal requests. 6. Safety concerns: If the statements have been dangerous or alarming, this goes beyond free speech and into potential threats or harassment. When crafting your response, consider addressing these points: 1. Acknowledge the importance of free speech, but emphasize its limitations when it infringes on others' rights and safety. 2. Highlight that the court order is a legal requirement, not a mere suggestion. 3. Explain how the defendant's actions have impacted your life, safety, and well-being. 4. Provide examples of the false or defamatory statements, if possible. 5. Mention the ignored cease and desist letter as evidence of the defendant's disregard for your rights. 6. Emphasize that as a private citizen, you have a reasonable expectation of privacy. It would be advisable to consult with a lawyer specializing in stalking or defamation cases to help you craft a strong, legally sound response. They can ensure that your arguments are presented effectively and in compliance with local laws and court procedures.

Justia Ask a Lawyer is a forum for consumers to get answers to basic legal questions. Any information sent through Justia Ask a Lawyer is not secure and is done so on a non-confidential basis only.

The use of this website to ask questions or receive answers does not create an attorney–client relationship between you and Justia, or between you and any attorney who receives your information or responds to your questions, nor is it intended to create such a relationship. Additionally, no responses on this forum constitute legal advice, which must be tailored to the specific circumstances of each case. You should not act upon information provided in Justia Ask a Lawyer without seeking professional counsel from an attorney admitted or authorized to practice in your jurisdiction. Justia assumes no responsibility to any person who relies on information contained on or received through this site and disclaims all liability in respect to such information.

Justia cannot guarantee that the information on this website (including any legal information provided by an attorney through this service) is accurate, complete, or up-to-date. While we intend to make every attempt to keep the information on this site current, the owners of and contributors to this site make no claims, promises or guarantees about the accuracy, completeness or adequacy of the information contained in or linked to from this site.

  • Bankruptcy Lawyers
  • Business Lawyers
  • Criminal Lawyers
  • Employment Lawyers
  • Estate Planning Lawyers
  • Family Lawyers
  • Personal Injury Lawyers
  • Estate Planning
  • Personal Injury
  • Business Formation
  • Business Operations
  • Intellectual Property
  • International Trade
  • Real Estate
  • Financial Aid
  • Course Outlines
  • Law Journals
  • US Constitution
  • Regulations
  • Supreme Court
  • Circuit Courts
  • District Courts
  • Dockets & Filings
  • State Constitutions
  • State Codes
  • State Case Law
  • Legal Blogs
  • Business Forms
  • Product Recalls
  • Justia Connect Membership
  • Justia Premium Placements
  • Justia Elevate (SEO, Websites)
  • Justia Amplify (PPC, GBP)
  • Testimonials

IMAGES

  1. A Guide to Answering Law Essay Questions

    answering essay questions in law

  2. How to Answer Law Essay Questions on an Exam

    answering essay questions in law

  3. Tort law How to answer questions

    answering essay questions in law

  4. Guide to Writing a Legal Problem Solving Answer

    answering essay questions in law

  5. Answering law questions

    answering essay questions in law

  6. How To Answer Law Exam Questions Perfectly (step By Step With Examples

    answering essay questions in law

VIDEO

  1. How to present theory papers (Law) in CA Exams. CA Shanil Usman

  2. How to get a First Class in your exams?

  3. How to create moodle essay questions

  4. How to Answer Economic Questions Part II

  5. 8-Mark Legal, Ethical, Cultural, Environmental Question

  6. ClassMarker Answering Essay Questions

COMMENTS

  1. How to Effectively Answer Law Essay Questions

    To answer law essay questions properly, it must follow four rules. It must have The Introduction, The definitions, the body, and the conclusion. Answering Law Essay Questions Rule 1: The Introduction. The introduction to your law essay question is the part where you let the lecturer know what the answer is all about.

  2. PDF Law School Guide to Answering Essay Questions

    The purpose of this guidance. This guide is intended to serve a baseline level of guidance for approaching essay questions in any subject offered by the Law School as part of your undergraduate law degree. It cannot, and does not purport to be, exhaustive. It is structured in a way which reflects the essay writing process and the main parts of ...

  3. PDF Guidance on how to answer problem questions

    Alongside writing essays, answering legal problem questions is likely to be one of the primary ways you are assessed while doing a law degree. A problem question usually takes the form of a factual scenario describing some event or events that have occurred. You are asked to 'sort out' this scenario in a legal

  4. How to Write Law Essay Questions

    The number one rule of answering any law essay question is RTFQ - ATFQ. Read the flaming question, answer the flaming question. Here is a common complaint by law examiners every year. Many students read the question once, see its on a particular topic, and regurgitate everything they know about that topic. As a result, the student fails to ...

  5. How to Answer Law Essay Questions (Law Lecturer's Guide)

    Pick the relevant cases & statute sections. The next thing you need to do in order to answer a law essay question is pick the relevant cases and statute sections to be included in your essay. I know - this seems very simple and straightforward, but this is where most students get stuck. The problem is you often come across an essay question and ...

  6. How to "Critically Evaluate" and "Analyse" in Law Essays (Law Tutor's

    In order to critically evaluate a law essay, you need to go beyond asking the basic questions. Just answering with the specific law on a certain issue isn't enough to make you a good lawyer. You need to build the ability to think for yourself and have an opinion on every case and statute, which you can defend with solid arguments.

  7. Answering Essay and Problem Questions

    As a law student, you will be expected to discuss these disputes and explain how they are likely to be resolved, if possible. The best approach to answering problem questions is the IRAC method, which stands for issue, rule, application, and conclusion. This is a four-step process that requires you to identify the legal issue; identify the ...

  8. How do I answer law school exam questions? (an in-depth guide)

    Here, we give you an in-depth guide on how to answer law school exam questions. This is a step-by-step process for approaching law school exams. Law school exams are very different than exams you may have taken in undergrad. Because they are different from the exams you are used to, you need a novel strategy if you want to answer the law school ...

  9. How to Answer Law Essay Questions

    If you're struggling to answer questions in your law essays, the advice in this video will point you in the right direction.-----...

  10. Writing Law Problem Questions

    This guide will explain how to answer a problem essay with eight handy tips. 1. Read the Facts. The first step to answering any law problem question is to read the entirety of the facts you are given. Do not just jump into answering the question. Take your time and ensure that you fully understand all the issue involved in the case.

  11. PDF How to Answer Fact Based Law Exam Questions

    T ypes of Examination Questions There are several types of law exam questions: short answer , essay questions, fact based questions, etc. Short answer questions typically require a few sentences by way of responses and assess recall of information. Essay questions test a candidateÕ s familiarity with and an ability to

  12. PDF HINTS AND TIPS ON ANSWERING EXAM QUESTIONS

    The essay question is asking you about the significance of Dica. Weaker answers will simply set out the facts of Dica and Konzani without offering much in the way of critical analysis or answering the specific question on the exam paper. Those answers usually show little (or any) understanding of the reasoning in Dica. Such answers will typically

  13. IRAC: How to Answer Law Problem Questions

    Just like we did in the guide to answering law essay questions, I am going to give you a sample question that we will use IRAC for. This is the question: Mr Daniel Kiss is a seasoned Fuji musician, he heard of the 10th year wedding anniversary of his long-time friend, Mr Starboy, who wanted to celebrate it in a big way. Mr Daniel Kiss was ...

  14. How To Answer Law Exam Questions Perfectly (step By Step With Examples

    How to answer essay-type questions in a law exam. In Answering essay-type questions the candidate will be required to write an essay. An essay consists of three major parts that are an introduction, a main body, and a conclusion. In order to successfully answer questions in law exams; a candidate must exactly know what the question demands.

  15. Law: Legal essay

    There are a number of strategies that may help you in starting, structuring and presenting a law essay. 1. Starting your answer. The first step to a successful law essay is understanding the question. One of the most effective ways of breaking down the question is to identify the direction, content, and scope or limiting words.

  16. Writing a Law essay? Remember to argue!

    Providing advice in the abstract about how to write Law essays is difficult because so much depends on the nature of the question you are answering. It's also important to take into account whatever are the expectations for your particular course, degree programme or university. Nevertheless, a useful rule of thumb, I think, is that…

  17. How do I answer problem questions?

    The best way to answer problem questions is to learn the technique. Examiners will expect you to answer problem questions in a particular way. Like with any type of exam, honing your technique is your best bet for problem questions too. In the case of problem questions, it's all about learning to apply your knowledge to the scenario.

  18. Problem Question Help Guide

    Free Law Problem Question Answer Guide. Example of how to answer problem questions from the experts at Law Teacher. ... Essays, case summaries, problem questions and dissertations here are relevant to law students from the United Kingdom and Great Britain, as well as students wishing to learn more about the UK legal system from overseas ...

  19. How to Answer Law Essay & Problem Questions Perfectly

    How to answer law questions: Over the years, law schools and law universities have evolved two methods of asking test or exam questions. They are the problem and essay questions patterns. Basically, these two ways of asking questions require a totally different ways of answering them that are different from the traditional ways of answering questions in the primary, secondary, and tertiary ...

  20. Bar Exam Essay Questions With Answers

    Passing the bar exam requires more than just knowing the law. Passing the bar exam also requires the ability to properly analyze a bar exam question, organize the issues, and write an effective answer. To demonstrate essay exam proficiency and how to apply the essential test-taking skills, consider the following bar exam sample question below, which will be used as an example to demonstrate ...

  21. A Handy Template for Answering "Hard" Law School Exam Questions

    Any time you're feeling panicked, you can fall back on this template. The most important element is #3: "Defendant will argue…" then "Plaintiff will argue….". If you can make both sides of the argument clearly, with reference to the appropriate legal and factual ambiguities, your conclusion hardly matters.

  22. 6 Top Tips For Answering Problem Questions In Law

    Here are some top tips on how to approach this type of question. 1. Read the question carefully. OK, so this really applies to all types of assignments, but with problem questions there can be a pretty lengthy scenario for you to get to grips with. It can help to highlight or underline, but even better try making a flow chart or chronology of ...

  23. Tackling Problem Essay Questions in Law

    The formula for tackling a law/legal problem question is as follows: Offer a brief introduction identifying the relevant area of law and any major legislation or cases that will be relevant. Identify relevant issues - do not repeat the question or the facts. Identify relevant legislation and/or case law (use only one or two relevant cases for ...

  24. Biden gave debate performance Democrats feared, but Trump did not win

    The question may be less whether Biden's showing Thursday will drive voters to Trump, however, or whether it will boost voter apathy, driving them stay home, to cast a ballot for Robert F ...

  25. Fact-checking Biden and Trump's claims at the first debate

    During a lengthy answer to a question about whether he would accept the result of the 2024 election and say all political violence is unacceptable, Trump made several false statements, including ...

  26. Advice on Feasibility of a Lease-to-Own Agreement for Real Estate

    Read 2 Answers from lawyers to Advice on Feasibility of a Lease-to-Own Agreement for Real Estate Investment - California Real Estate Law Questions & Answers - Justia Ask a Lawyer

  27. how can i add my new born baby to I-131

    A: To add your newborn baby to your pending I-131 application (Application for Travel Document), you should follow these steps: 1. Contact USCIS: Reach out to the U.S. Citizenship and Immigration Services (USCIS) as soon as possible to inform them about your new situation.

  28. Please let me know if a first position lender can revoke a loan if a

    A: To answer this question accurately, we need to consider a few key points about mortgage lending practices and California law: 1. Loan agreements and due-on-encumbrance clauses:

  29. Revealed: Harvard Business School's New MBA Essays For Applicants

    With just 10 weeks before its first application deadline on Sept. 4th, Harvard Business School today (June 25) revealed a newly revised application for MBA candidates, including a new set of three short essays along with a refresh on how it will evaluate applicants for future classes. The new prompts? Business-Minded Essay: Please reflect on how your experiences have influenced your career ...

  30. I have filed a Protection From Stalking order in the State of Kansas

    Read 1 Answer from lawyers to I have filed a Protection From Stalking order in the State of Kansas and a temporary order has been granted. - Kansas Family Law Questions & Answers - Justia Ask a Lawyer